Sunteți pe pagina 1din 458
Peet } ~ Cc Ce a Pr ae cm SS ier TS TN ry ro hs aan q,,)_ oh ee Bditorial AMERICA a ae MATEMATICA BASICA 2 VECTORES Y MATRICES Primera Edicion. Marzo 1985 Segunda Edicién: Marzo 1988 Reimpresion de la Segunda Edicion. Agosto 1990 Agosto 1992 Agosto 1993 Impreso por : EDICIONES E IMPRESIONES GRAFICAS AMERICA S.R.L. Jr. Loreto Nro. 1696 Brefia (Lima 5). Telefax 325827 Revisado por: RICARDO FIGUEROA GARCIA Egresado de la Universidad Nacional de Ingeneria Facultad de Mecanica Todos los derechos reservados conforme al Decreto Ley Nro 19437 Queda prohibido la reproducci6n por cualquier medio. total o parcialmente, sin permiso escrito de! autor. JIT U a oe r © » eo Dada la acogida que le dispensaron los estudiantes a las edi- ciones preliminares de esta obra, explica la aparicién de esta nueva edicién ampliada, en la que se han hecho las modificacio- nes necesarias con el propdédsito de hacer m&s asequible su lectu- ra, pues la obra proporciona una excelente preparacién para el estudio de cursos superiores como el Andélisis Matematico y sobre todo, el Algebra Lineal. El estudiante que ha llegado a este curso ya tiene conocimien- to del Algebra y la Geometria Elemental. En el primer capitulo se desarrolla la relacién que existe entre estos dos grandes cam pos de la matemética; esto es, el estudio de la técnica de los vectores. Los sistemas de coordenadas que se utilizan, primero el bidimensional (plano) se extiende después al tridimensional (espacio), indicando claramente el camino para generalizar los conceptos a otras dimensiones, y luego finalizar, haciendo un breve estudio de los espacios vectoriales. En el segundo capftulo se hace referencia al estudio de las ma trices de acuerdo con su dimensién o tamafio y sus aplicaciones a la solucién de ecuaciones lineales En el tercer capitulo se expone la teorfa de los determinantes, de particular importancia en la teorfa de las matrices y sus nu- merosas aplicaciones. Con este libro se tiene la intensién de desarrollar la capaci- _. dad del estudiante y crear en él hdébitos de rutina matematica; esto es, la exposicién teérica es acompafiada de numerosos ejen- plos y ejercicios con sus respuestas adjuntas, los cuales, indu- dablemente, ayudardn al estudiante a adquirir destreza y afirmar el dominio de la materia. Por ello, recomiendo que los ejercicios propuestos se resuelvan sistemdticamente, toda vez que su solu- cién obedece a un criterio de aprendizaje progresive. IV Préblogo Mi reconocimiento a todos los amigos profesores que tuvieron la gentileza de hacerme llegar sus sugerencias y observaciones a las ediciones preliminares. Sus criticas constructivas hicieron posible corregir, mejorar y ampliar esta nueva edicién. Ricardo Figueroa Garcia 4.42 4.18 4.19 1.20 1.23 4.24 425 4.26 1.27 128 1.29 136 CONTENIDO VECTORES Introduccion. 4.2 Coordenadas Cartesinas Vectores en el plano. Representacion geomeétrica de un vector. Magnitud de un vector. Propiedades. Direccién de un vector en R?- Vector Unitario. Adicion de Vectores. Propiedades Representaci6on grafica de la adicin de vectores Sustraccion de vectores. Multiplicacion de un escalar por un vector. Representacion grafica. Propiecades. Vectores Paralelos. Producto escalar de vectores. Vectores ortogonales. Angulo formado por dos vectores. Descomposicién de vectores. Proyeccién Ortogonal. Componentes Escalares. Area del paralelogramo y del tnangulo. Descomposicion Lineal. 7.21 Independencia Lineal. 4.22 Criterio de Independencia Lineal. Regla de comparacién de coeficientes. Aplicacion de ios vectores a la Geometria Elemental. Aplicacion de los vectores a la Fisica. ECUACIONES VECTORIALES DE LA RECTA Rectas en el piano. Segmentos de recta Divisién de un segmento en una razon dada. Puntos que estan sobre una recta. Pendtentes de una recta Rectas paralelas y ortogonales. 107 108 110 415 120 VI 1.31 1-32 1.33 1.34 1235 1.36 1.37 1.38 1.39 4.40 1.41 1.42 1.43 Contenido ECUACIONES CARTESIANAS DE LA RECTA Forma general de la ecuacién de una recta. Forma Punto-Pendiente. Forma Pendiente y Ordenada en el origen. Forma abscisa y ordenada en el origen. Forma Simétrica. RELACIONES ENTRE RECTAS Distancia de un punto a una recta dada. TInterseccién de rectas. Angulo entre rectas. EL ESPACIO TRIDIMENSTONAL VECTORES EN EL ESPACIO Direccién de un vector en R*. Vectores Paralelos y Perpendiculares Proyeccion Ortogonal. Componentes. Combinacién Lineal. 1.44 Dependencia e Independencia Lineal. Base y Coordenadas de un vector en R°. EL PRODUCTO VECTORIAL Propiedades del producto vectorial. Interpretacién geométrica del producto vectorial. PRODUCTO MIXTO DE VECTORES. Propiedades e interpreta- cidn geométrica,. RECTAS EN EL ESPACIO. Posiciones relativas de rectas en el espacio, Distancia de un punto a una recta. Distancia entre dos rectas en el espacio. PLANOS EN EL ESPACIO, Ecuacién vectorial del plano. Distancia de un punto a uf plano. Interseccién de planos. Angulo diedro entre dos planos. 1.59 Angulo entre una recta y un plano. Proyeccién ortogonal de una recta sobre un plano. 128 130 134 132 132 135 141 TA9 159 160 167 170 177 181 182 187 189 192 201 209 212 217 219 223 224 R29 233 237 238 Contenido 1.64 4.62 1.63 164 1.65 1.66 1.67 8 2.11 2.12 2.13 2.14 2.16 2.19 2.20 2.21 2.22 2.23 VII Transformaci6n elemental fila. Matriz Escalonada Matrices Equivalentes. Rango de una Matiz. Niatrices Elementales. INVERSA DE UNA MATRIZ por el método de Interseccién de rectas y planos 241 Vecicres Je n dimensiones. 251 ESPACIOS VECTORIALES. 25 Subespaciss vectoriales. 258 Indeperdencia Lineal. 264 Bases y dimensiones de un espacic vectorial 269 Suma de subespacios. 276 MATRICES Intreduccién. 2.2 Definici6n. 281 Orden de una matriz. 282 Tipos de Matrices. 283 Igualdad de Matrices. 284 Suma de Matrices. Propiedades. 285 Diferencia de Matrices. 286 Productc de un escalar por una matriz. Propiedades. 286 MultiplicaciOn de Matrices. 289 Propiedades de la Multiplicacion de Matrices 293 MATRICES CUADRADAS ESPECIALES Matriz Simétrica. 305 Matriz Antisimétrica. 306 Matriz Identidad. 307 Matriz Diagonal. 2.15 Matriz Escalar. 309 Matriz Triangular Superior. 2.17 Matriz Triangular Inferior. 218 Matriz Periddica. 310 Matriz Transpuesta. 374 Matriz Hermitiana. 316 MATRIZ INVERSA 317 Inversa de una Matriz Triangular. 319 TRANSFORMACIONES ELEMENTALES. 327 NHI 2.24 2.25 2.26 3.7.2 3.7.3 3.7.4 3.7.5 3.7.6 3.7.7 3.7.8 3.7.9 Contenido Gauss-Jordan Sistemas de Ecuaciones Lineales Rango de un Sistema de Ecuacioneés Lineales. Sistemas Homogéneos de Ecuaciones Lineales DETERMINANTES Definicién. Propiedades. Existencia de los Determinantes. Menor de una componentes. Cofactor de una componente. Calculo de determinantes de cualquier orden. Otras aplicaciones y Propiedades de los determinantes. 3.7.1 Regla de Sarrus. Caiculo de determinantes mediante reduccién a la forma escalonada Propiedades Muktiplicativas. Rango de una Matriz. * Adjunta de una Matriz. Inversa de una Matriz. Matrices no singulares. Resolucién de sistemas de ecuaciones de dos variables. Resolucién de sistemas de ecuaciones en tres variables. 3.7.10 REGLA DE CRAMER. 343 354 359 367 368 375 376 377 381 401 402 412 416 422 424 436 444 442 443 rs rrr cry er || VECTORES \ 1.1 INTRODUCCION Hace muchos afios los griegos deserrollaron la geonetria elerental. Crearon una manera siste mAtica de analizar las propiedades de los puntos, lias rcoctas, los trigngulos, las circunferencias y otras configuraciones. Todo su trabajo fué sintetizado en "Los elementos de Euelides" , que hen constituido las bases de la geometria plana y del espacio hasta nustros dias. En tiempos recientes, se han agregado otros conjun- tos de axiomas y postulados, cuyo efecto han sido mejorar le es- tructura légica, pero, en esencia, la materia ha permanecido idén tica. En 1637, el filésofo y matemético francés René Descartes re volucioné 1a matemética de su époea al crear la Geometrfa Analiti ca introduciendo las coordenedas rectangulares, llamadas también en su memoria, coordenadas cartesianas; logrando asi elgebrizar las ideas geométricas de sus antecesores. La idea de este método eonsista en traducir, meGiante un sistene ¢e coorcenadas, los con ceptos y relaciones geonétricos a conceptos y relaciones algebrai cas, y viceyersa. En este capitulo estudiaremos el método anliti- co pera lo cual precisamos familiarizarnos con el concepto de vec tor, un instrumento de gran valor en la matem4tica moderna. 1.2 COORDENADAS RECTANGULARES En estudios anteriores de mateméticas definimos el producto cartesiano AxB, de los conjuntos A y 8, como el conjunto de todos los pares ordenados (x,y) en los cuales la primera componente xX , es elemento de A y la segunda componente y, es elemento de B. Por ejemplo, si A={2,3,5} y B={1,3}, entonces: AXB = ((2,1), (2.3), (3.1), (3,3), (5,1), (5, 3)] Un conjunto de peres ordenados AxB se puede visualizar como una red cde puntos, tal como se indica en la Figure 1. 2 Vectones 4 ~ ~ Come los pares ordenados de numeros reales scn elementos del gre j 2 dueto cartesiano RxR, a este conjunto se le denota ror K*, es de R2 = RXR = ((x,y)/xeR , yeR} | a | | | PJ y 34- srr ee Figura 1 Figura 2 Obsérvese, en la Figura 2, que cada par ordenado (a,b) en R* se puede asociar en forma Gnica con un punto P del piano mediante un sistema de coordenadas rectangulares, al que se llama también séstema de coondenadas canteséane. El asociar a cada par ordenado (a,b) un punto P se lleva a cabo como sigue: a) Por un punto que corresponde al nfimero a sobre el eje horizon- tal (eje de abscisas) se traza una recta paralela al eje verti cal. b) Por el punto que corresponde al nimero b sobre ei eje vertical (eje de ordenadas) se traza una recta paralela al eje horizon- tal. c) Al punto de interseccién P de estas rectas se le asocian las coordenadas (a,b). P se llama "la gréfica de (a,b)" 0 simpie- mente "el punto (a,b)". En adelante, a los elementos de R? los denotareamos con letras mayisculas: A,B,C, etc. Por ejemplo: A=(a2,a2), B=(by,b2). DEFINICION 1. Dados dos pares ordenados A=(a,,a2) y B=(by,b2) en R?, la suma de A y B, denotado por A+B, esté defi- nido por: —————————_—— rr TT RE Vectores 3 A+B = (a1,82)+(bi,b2) = faitbi , aztb2) Se puede observar que la adicién de dos pares ordenados de nime- ros reales es otro par erdenado de nimeros reales. Por ejempio, si A=(2,-5) y B=(2,3), entonces: AHB = (2,-5)4(2,3) = (2+2,-543) = (4,-2) DEFINICION 2, Dado un numero real r, llamado escalar y el par or denado A=(a1,42), se denomina producto del escaler r por A, al par ordenado: rA = r(ai,az) = (rai,raz2) Obsérvese también que rA-R?. Por ejemplo, si r=~2 y A=(-1,3), entonces: j enp y rA = -2(-1,3) = [(-2)(-1), (-2)(3)] = (2,-6) PROPOSICION 1.1 Dados los pares ordenados A,B,CeR* y los escala- res r,seR, se cumplen las siguientes propiedades para la adicién de pares ordenados y la multiplicacién de escala- res por pares ordenados: Ait Si A,BeR? ~+ (A+B)eR? (Clausura) Az: Si A,BeR? + A+B = BHA (Conmutatividad) As: Si A,B,CeR? - (AtB)+6 = A+(BtC) (Asociatividad) Ag: 3!6eR7/AtO = O+A = A, ¥AER* (Elemento identidad pera le adicidn de pares) Pi: Si reR y AcR? -> rAeR? Pe: r(AtB) = rAtrB , ¥reR , ¥A,BeR? Ps: (rts)A = rAtsA , ¥r,seR , ¥AcR? Pe: (rs)A = r(sA) , ¥r,seR , ¥AcR? Ps: 31eR/1A = A , ¥AcR? As: ¥AcR*, 3!~AcR?/aAt(-A) = (-A)#A = 6 (Elemento inverso vara le adicién de pares) Se recomienda al lector demostrar cada una de estas propiecades haciendo uso ce las prcpiedades respectivas de los nimeros reales. 4 Vectonres Bl conjunto R? de pares ordenados de nimeros reales, junto con las operaciones de suma y producto definideas anteriormente recibe el nombre de espacio vectorial kidimensional sobre el conjunto de los némeros reales R y se denota por V2. A los elementos de un es pacio vectorial se les llama vectores; por tanto, podenos afirmar que el par ordenado (x,y) es un vector. 1.3. VECTORES EN EL PLANO Un vector en el plano es un par ordenado de nimeros reales (x,y}, donde x recibe el nombre de primera componente (coordena- da) e y se llama segunda componente. A los vectores en el plano se les denota por letras minfésculas o mayfscules con una fleche en la parte superior. Por ejemplo: 2 , 4 , z , k ’ B » etc, Dado dos vectores en V2: a=(xasyn) y Bb=(xasy2)) podenos definir Xi = Xz i)Sia=6b + (Igualdad de vectores) yy. = y2 i) 242 ii) a+ b= (xitxa » yaty2) (Def. 1) aii) ra = (rx.,Ty1) (def. 2) femplo 1. Si a=(-2,3) y b=€4,-1), hallar el vector y=2at3b. 2(-2,3) + 3(4,-1) . > Solucién. v = (-4,6) + (12,-3) (Def. 2) = (-4412 , 6-3) (Def. 1) = (8,3) Ejemplo 2. Hallar el vector x en la ecuacién: 2(~1,2)+3x=(4,-5). Solucién. Supongamos que: x = (x2,x2) + 2(-1,2) + 3(x1,x2) = (4,-5) > (-2,4) t (3x1, 3x2) = (4,-5) (Def. 2) + (-24+3x1 , 443x2) = (4,-5) (Def. 1) Por la igualdad de vectores se tiene: ~24+3x2 4h > x1=2 At3x2 = -5 ++ x2=-3 Por tanto, el vector buscado es: x = (2,-3) u — TE AA Vectones Ejemplo 3. Haller todos los nimeros reales r y s tales que: r(4,-6) + s(5,-2) 5 (7,6) Sofucién. (4r.-6r) + (5s.-28) = (7,6) (Def. 2) (Art5s , -6r-2s) = (7,6) (Def. 1) Por la igualdad de vectores: 4rt5s = 7 -6r-28s = 6 Resolviendo el sistema obtenemos: r=-2 , s=3 1.4 REPRESENTACION GECMETRICA DE UN VECTOR EN EL PLANO Geométricamente un vector v=(x,y) se representa en el plano mediante un segmento de recta dirigido o una flecha. La flecha se llama vector geométnico. Un vector veR? puede interpretarse como una traslacién descrita por un par ordenado de nimeros reeles (x,y), la primera componente indica un desplazamiento paralelo al eje X vy la segunda un desplezamiento paralelo al eje Y. Considerando que una treslacién tiene un punto inicial o de panty da S del plano, y un punto final o de llegada en T, cada vector v=(x,y) tiene un nimero infinito de representaciones geométricas en el plano, todas ellas son paralelas, de igual longitud e igual sentido. (Figura 3) ~1_.6 —4-w ge. ' 8, ' t nT LA t 1 ' ee -p---4-- ~+-do gy -b- we t-Ag r-t-- t + t Figura 3 La flecha asociada al par (x,y) que tiene un punto inicial en el origen se denomina rzepzzesentacién ondinania de (x,y) y se dice que la flecha o vector tiene posicién ordinaria o estandard. teredoaes g DEFINICION 3. VECTOR LOCALIZADO Un veeter lecalizado ex 27 es una pare’a de puntos P: y Pa que se indican con PiP2 para los cuaies F, es el punto de partida o inicial y Pz es el punto de llegads s finai (Figura 4). Si una flecha tiene como punto inicial a Piiyia,yi) y a Palxe,y2} como punto final, entonces 1a fiecha PiF2 es una representacién geométrica del vector v=(x,y), donde: (x,y) = Gca-xas yarya) (1) Si consideramos a los puntos P; y Facomo radio vectores entonces, segin la definicién 3: “ —> = P}P, = B,-B, > Vv > B, = 3, + z (2) Esta ecuaci6n nos peraite conocer analfticamente el punto final Pz del vector ? conociendo, desde luego, el punto inicial y las componentes del vecor ve DEFINICION & VECTOR DE POSICION Todo vector que tiene posicién ordinaria, es decir, al vector que tiene su punto inicial en el crigen se llama vectoa de posicéién o aadio vectoa. Observaciones: 1. El vector localizado PiP: es équivalente al vector de pcsi- cidn v=F2-Fi. La ley del parieiogramo hace evidente esta equi valencia. (Figura 5) 2. La notacién P(x,y) identifica un punto en ei plano y sus coor denadas (x,y) identifican a un vector o a su representacién geométrica. Figura 4 Figura 5 Vectorzes Ejemplo 1. Hallar el vector de posicién de PiP2 si Pi(5,-2) y P2(2,3). Interpretar geométricanmente el resultado Solucién. Segin la definicién 3: v = PiP2 = B,-B, (2, 3)-(5,-2) (2-5, 3+2) (-3,5) Ejemplo 2. Un vector que va de R(3,5) a S(x,y) representa al mis mo vector que va de S(x,y) a T(8,1). Haller S(x,y). Solucién. Sean: & = RS = 8 - ® = (x,y)-(3,5) = (x-3,y-5) $ = St = B- 8 = (8,1)-(x,y) = (8-x,1-y) . x-3=8-x > x=11/2 Si a=b + (x-3, y-5)=(8-x, 1-y) + y-5=1-y + y=3 Por tanto, el punto buscado es: S(11/2,3) Ejemplo 3. En la figura adjunta se tiene: OP=x3 y 0Q=x2y. Si 2=b, siendo b=(y#+19,6+xy?). Hallar el valor de xty. Solucién. Las componentes del vector a son OP y 0Q + a=(x*,x?y) + x8 = y3t19 + xF-y3s19 (1) Luego, si asb + fay = 6txy? + x?y-xy?=6 (2) Multiplicando por 3 la ecuacién (2) y restando de (1) se tiene: x3. 3x? y4+3xy?-y3=1 + (x-y)3=1 , de donde: x=yt1 (3) Sustituyendo (2?) en (1) obtenemos: y2ty-620 ++ y=-3 6 y=2 Descartamos la segunda alternativa ye que en la figura dede, OP es negativo. Luego, en (3): x=-3t1=-2 o. xty=-5 Vectiores EJERCICIOS Dados: a=(3,-4), b=(8,-1) y @=(-2,5), hellar el vector ¥ si: > 2) ¥ = 3a - 26+ Rp. v=(-9,-5) b) 3 = ub + (6-8) Rp. ¥=(17,-19) ce) ¥ = 2(2-B) + 38 Rp. ¥=(-16,9) Heller el vector x en las siguientes ecuaciones: a) 3(0,-2)+2%-5(1,3) = (-3,-5) Rp. x=( b) (18,-12)42 (-6,5)#% = 4(15-2) Rp. x=( =n las siguientes relaciones hallar, si existen, todes los nimeres reales ry s. a) r(~2, 3)-s(8,1) = (16,15) Rp. r=4, s=-3 b) r(5,1)+s(-3,5) = (-2,8) Rp. r=1/2, s=3/2 ec) r(-2,3)+s(4.-6) = (0,2) Rp. dr,s Dedos los vectores a=(3x-5,x-2y+2) y b=(x-y-2,3-2y), haller x @ y Ge modo que: 3a=4b Rp. x=5, y=-¢/2 Si @=(2n-3n,4n-m) y b=(2,-3), hallar los valores de n yn que hacen que: a=50. Rp. m=-1, n=-4 41 vector ¥=(3,2) es el vector de posicién del segmento AB, cuy® punto medic es C(3,1). Hallar les coordenedas de los extrenos del segnento AB. Rp. A(3/2,0), B(9/2,2) Sean los puntos P(5/2,5), Q(1/3,13/4), R(-16/5,7/2) y S(x,y) si FQ y RS repréesentan al mismo vector, calcular ¢1 valor de 30x+80y Rp. -21 Sea v=(7,-6) el vector de posicién del segnento AB y c(2.3 el punto de triseccién més cercano de B, de dicho nic Hallar las coordenadas de A y 2. Rp. A(-3,7), B(4,17) Sean A(a,-2), B(2,4), 6(8,-3) y D= (x,y)/y=2x+1 . Si AB=CD, haliar el valor de a-x. Ro. 8 -.. fa figura adjunta se tiene: OP=x3 y 0Q=6-x Faller @, si b=(9xy-y3,y) y 2-8. om sO Vectores 1.5 MAGNITUD DE UN VECTOR > + . Para cada vector veR?, v=(x,y), existe un escalar o nimere . > + llamado noama, médulo o magnitud de v, denotado por ||v{], tal que: II] | = “x+y? (3) y4 La férmula (3) es coircidente con la nocién intuitiva de longitud de un segmento derivada del Teorema de Fi- tégoras. La Figura 6 ilustra esta pro piedad, Figura 6 EjJemplo 1. Hallar la magnitud del vector de extremos A(1,3) y B(-2,7). Solucién. Si ¥ es el vector que va de A a B, entcnces: v = AB = B-K = (-241,7-3) = (-3,4) Luego, segtin (3): LUI = /(-3)#4#(4)? = 5 PROPIEDADES DE LA NORMA DE UN VECTOR EN R?2, Ni: ¥aeR? , [fa] |>o ( Na: [Jal[=0 ++ & =e Nat ¥reR , ¥aeR7, []ral{ = Ir] 18] Ny: ¥a,beR?, [Jarl] <[]al] + [Jbl] (Deeigualdad triang.) Demostracién de Ny: En efecto, si a=(x,y) > [lal] = “e2ty? Si x#0 © y#0 + [fal] # 0. Sabemos que si existe la raiz cuadrada de un nimero, esta es positiva, por lo tanto, |/a][>0. Demostracién de N2: (>) si @-6 + 2=(0,0) + [lal] = “0240? = 0 (+) Si |Jal[=O +, [Jal] = %?2+y? = 0 . La igualéad es vali si x=y=0, esto es, a=(C0,0)=6. .%. |Jal|=0 ++ a=6 + 40 Vectcnet Demostracién de Ng: En efecto, si a=(x,y) * vra=(rx,ry) y [ral l = Alex) 2e(ry)? = Ae2(xtty?) = oe? ARP ay? > > Por consiguiente: [|ra[] = Irl-flal] 1,6 DJRECCION DE UN VECTOR EN R?, > : A cada vector no nulo, v=(x,y)eh”, le corresponde una direc ; . > ciSn daca por la medida del énguio a (dngulo de direccién de v), que forma el vector con el semiejJe positivo de les X, para ei cual: Sena = —— = meee HIvil | %«P+y? . (4) Cosa = = = ivi x+y? y 0° < m({a) < 360°, De las ecuaciones (4) se sigue que: v= Oy) = [I] | (cosa, Sena) (5) Por tanto, un vector queda determinade por su magnitud y su di- receion. Observacién. La direccién m(a) del vecter Vv se obtiene de la ra nera siguiente: Mediante un d4ngulo de referencia a, y haciendo uso de una tabla ce valores se halle el valor de a; con 6°G , y>O + aa) = m(ay) (Cuadrante I) xO, y>O + ma) = 180°-z(a1)} (Cuadrante IZ) x<0 , y0 6 y0 y Cosa<0, entonces a est& en el II cuadrante. Angulo de referencia: Tgai = I-41 = $ + 01=53°8! Por tanto: m(a) = 180°-53°8! = 126°52! Ejemplo 3. Expresar el vector V=(3,-3/3) en términos de su nag nitud y de su Angulo de direccién. ——— Soluctén. Segtin (3): JI¥{} = “(3)2+(-373)?2 = 6 y por las ecuaciones (4): Sena -.% Cosa = 2 2 ¥ “° 2 Como Sena0, entonces a estaé situado en el IV cuadrante. Angulo de referencia: Tga) = I= | = V3 de donde: m(a,}=€0° + m(n)=360°-60°= 300° Por tanto, segin la ecvacién (5): ¥ = 6(Cos 300°, Sen300°) 1.7 VECTOR UNITARIO . + . . Dado un vector no nulo v=(x,y), llemamos vector unitario a > . . > un vector u que tiere la misma direccién de v para el cuel: > + v x y us = (——., ) (6) Livi i Hythe o bien: u = (Cosa , Sena) (7) Ejenplo 4. Haller un vector unitario que tiene la misma direc- eién y sentido del vector v=(-3,”7) Solueéén. Segin (3): [[¥|| = A-3)?24+(V7)? = 4 12 Vectores y por (6): f= 3). (. Bho ald — Ejenploe 5. Haller un vector de médulo 10, que tenga la misma Gireccién y sentido opuesto al vector gue va de S(4,2) a 7(1,6). Sotucién. Sea ¥=ST=4-S8=(1-4,6-2)=(-3,4) 2 a > Un vector unitario en le direccién de v es: > +) > u = £3,4) . Luege, el vector tuscacc es: v = -{ vila “. % = (6,-8) An t bh EJERCICIOS En los ejercicios del 1 8l 4, se Gan las coordenecas de los puntos A y B. Expresar cade vector V=AB en térninos de su magnitud y de su Angulo de Cireccién. 1 A(-3,4) , B(-5,6) R. ¥=272(Cos135°, Sen135°) 2. A(/T2,-3) , B(Y27,-4) R, ¥=2(Cos330°, Sen330°) 3. A(573,4) , B(TB,S) R. ¥=2(Cos150°, Sen150°) 4. A(345,-V75) , B(E0,-/60) R. v=2/E(Cos240°, Sen240°) 5. Hallar un vector ¥ cuya magnitud es igual a la del vector a=(4,-3) y cuya direccién es la misma que la del vector B=(1,73). = v3 Rp. ¥ = (3,204) 6. Hallar un vector de médulo 10 que forma un Angulo de 37° con el eje X positivo. (Sug. Cos37°=4/4) Rp. ¥=(8, +6) 7. 4Hallar un vector de nédulo 15 que forma un Angulo de 53° con el eje Y positivo. (Sug. Cos53°=3/5) Rp. ¥=(-12,9) 8. Hallar un vector que tenga la misma magnitud del vector que va de A(-2,3) a B(-5,4) y que tenga el sentido opuesto al vector que va de S(9,-1) a T(12,-7). Rp. ¥=/2(-1,2) 9. Hallar un vector ¥ de longitud 6/3 y que tiene la misma di- reccién de un vector que forma un angulo de 30° con el sen- tido positivo del eje X. Fp. v=(9,233) Vecioneas 13 OPERACIONES VECTORIALES 1.8 ADICION DE VECTORES EN EL PLANO Dados dos vectores a y $ en R? tal que a=(xa,ya) y b=(xa.ya); definimos la adicién del modo siguiente: atb = (xa,yr)4(x2,y2) = (xitxe,yaty2) Por ejemplo, si a=(5,-7) y b=(~3,2), entonces: a+b = (5-3,-742) = (2,+5) PROPIEDADES DE LA ADICION VECTORIAL. Si a,b y @ son vectores en R?, entonces se cum- plen las siguientes propiedades: Ai: (atb) eR? Clausura Aa? at+b=b+ a Conmutatividad As: (2 +3) + G58 + (B+ 3) Asociatividad Ay: d@eR? , VaeR?/A+o=0+a = & Elemento neutro para la adicién As: ¥aeR? , a(-a)eR?/a+(-f)=(-a)ta = 6 Opuesto de un vector Demostracién de Ai: En efecto, si a=(Xisyn) y b=(xe.y2)> entonces: athe (xitx2,yity2) (Def.1) Puesto que la adicién es cérrada en R > (xatxzJeR y (yity2)eR Por tanto: (xitx2,syityz2jeR? ++ (atb)eR? Demostracién de Az: Consta de dos partes: Existencia y Unicidad. Existencia. Si a=(x1,y1), se tiene: & + O= (xi+y2)4(0,0) = (x140,yit0) = Ga,ya) = 2 Andlogamentet @ + a = 2 Unicidad. Sea @, otro elemento de R* que tembién cunvle > > + a+t6; = @,;, taza 7 : : > : + Esta igualdad es cierta ¥aeR?, en particular si a=6, entonces: 14 Vectonres 6 + 6, = 6, + 6 = 6 : > . Andlogamente, haciendo a=6, en Ay se tiene que: 6, + 6 = 6 + 8) = By ne las dos iguatdades anteriores prueban que 6, = 6 *? he ° sO ‘ Se deja al lector demostrar las propiedades Az, Ag y As haciendo uso de las propiedades que cumple la adicién en R. 1.9 REPRESENTACION GRAFICA DE LA ADICION DE VECTORES EN EL PLANO Dados a y beR?, la flecha que representa a la suma RB se obtiene de la manera siguiente: Representamos una traslacién a lo largo de une flecha cualquiera que represente al vector 2=(x1,y1) seguida de una traslacién del punto final de esta flecha a lo largo de la flecha que represen- ta al vector b=(x2,y2). La traslacién total correspondiente al vector ath, es una flecha que tiene como punto inicial el del vector a y como punto final el del vector %. (Figure 7) (xitxe,yity2) Dp z ad igura En esta construccién los vectores a y b son lados adyacentes de un paralelogramo y la suma a+b es la diagonal correspondiente. La obtencién de la suma de vectores siguiendo este procedimiento recibe el nombre de fey def panalbetogaamo, que se ilustra en el Siguiente ejemplo. Vectores 15 Ejemplo 1. Dados los vectores a=(-1,4) y $=(3,2), hallar a+b ¥ construir una gréfica que muestre las representacio- nes ordinarias correspondientes a los vectores. Sotucién, Por definicién: atB = (-14+3, 442) = (2,6) Observemcs que la flecha que va de S a T representa al vector a y la fle- cha que va de R a T representa a b. (Por segmentos de paralelas) DEFINICION 5. NEGATIVO DE UN VECTOR EN R? Si 4eR?, tal que a=(x,y), se denomina negativo o inverso aditivo de @ al vector: “2 = (-x,-y) Por ejemplo, el negative del vector a=(-3,2) es -a=(3,-2) Observacién. Dado el vector aeR?, 3u negativo -aeR? es colineal, de la wisma magnitud: es > > . to es: |-al=|af, pero de sentido o > puesto que el vector a. 1.10 SUSTRACCION DE VECTORES Dados dos vectores a,bceR?, tal que a=(x1,y2) y b=(x2+y2), definimos la diferencia a-b del modo siguiente: a-b=a+ (-B) =" (xi.y1) + (-x2,-y2) . > or Aa $= (xi-x2,y1-y2) (8) Ejemplo 2. Si 4=(4,2) y B=(-3,3), hallar la diferencia a-b y tra zar una grafica que muestre la representacién ordina- ria de los tres vectores. (4,2)-(-3,3) = (4,2)+(3,-3) (443,2-3) = (7,~1) Svéucéén, Por definicién: a-b 16 Vecionres La repressntacién ordinaria de cada uno de ios vectores se muestran en la Figura @. Debemos destacar que, el inverso aditi- vo de (-3,3) es (2,-3) (negativo del vector 6) 5+ s gue (-23,3) vero de sent 2 que (-3,3) pero de sent y de . > + : La representacién geonétrice de a-6 puede obtenerse aplicando lea aa nismna 2 Risma ma we la regla del paralelogramo a la suma at+(-5). Le Figura 9 nos mu- + estra otra manera Ge representar la diferencia a-5. Figura & Figura 9 Observaciones: . > = . > 1. Sia, beR?, entonces ia diferencia 4-6 setisface la condicidn > + >~ $+(a-b)=a, to que explica porque algunas veces se dice que le diferencia a=b es el vector que v es el q adeba a. 2. El vector diferencia une los puntos finales de los vectores 3 y a (Figura 9). .- > 7 3. Sia, ber?, son vectores no nulos, entonces 4-> # $-a Ejemplo 3. Sea x un vector tal que (3,-2)=x+(1,-6). (3,~2)=tx+r(-1,1), hallar el valor de 3r+6t. Scéucién, En la primera ecuacién se tiene: (3,-4)-(1,-6) = % + (1,-6) - (1,-6) + (3-1,-446) = X + 8 (Ay) + (2,2) = & Luego, si (3,-2) 4(2,2)4+r(-2,1) + 3,-2) = (2t+2r,2t+r) Por igualdad de vectores: 3=2t+2r y -2=2ttr Resolviendo el sistema obtenemos: r=-5/3 y t=-1/6 . 3rt6t = -6 Vectonres 17 Ejemplo 4 Dados: 2=(-2,2), 6=(3,-2) y G=(-1,1), resolver la e- cuacién: 3a - 2[3(6-2¢) + 22] + 3x = 20+ x. Soéucéén. Restendo 2¢+x a cada extremo de la ecuacién dada se tiene: 32-6(B-2¢)-4a43x-(2etx) = (2e+z)-(2e+x) + -2-664+120+3%-20-% = 0 de donde: 2x = a+6b-10@ = (-2,2)+6(3,-2)-10(-1,1) (~24+784+70 , 2-42-10) (26,-20) (43, -10) a+ i Ejemplo 5. Mediante segmentos orientados demostrar la propiedad > > +> > As: (atb)+e = at(bte). Demostnacién, En efecto, sean los segmentos orientados: PT = 4,78 = 6, SR=¢, PR = x Por la interpretacién gréfica de la suma de vectores se tiene: 4 s En el APTS: PS =PT+TS =atb t En el ATSR: TR=7S + SR= B40 z 3 En el APSR: PR = PS + SR > x= (446) +e@ (1) En el APTR: PR = PT + TR P : R > > > mn + x=a+ (b+) (2) Por tanto, de (1) y (2) se sigue que: (246) 42 = 244642) Ejemplo 6. Sean a=(-2,3) y t=(4,-3). Un segmento dirigido que representa a (3a - 26) tiene por punto inicial 8(5,-3/2); hallar el punto final. Soflucién. Sea T(x,y) el punto final del segmento ST. _ am. 2t i ~2 4 Si ST = $a - 5 > #F-8 = 3(-2,3) - (4,-3) 3 5 x-5 = -2 +> x=3 Entonces: (x-5.y + $) = (-2,5) ome yt3/2 = 5/2 + y=1 Por tanto, el punto final es: T(3,1) 18 Vectores Ejemplo 7. Se tiene: 2(2,-3)te = (3,-5)+{a,7) y @ esté sobre la recta L:y=x+2. Si-A(3,5) y B(-2,6), hallar el punto P tal que PC = -AB. soducién. Si teL + t=(x,xt2) , + 2(2,-3) + (x,x#2) = (3,-5)+(at7) = a-1 / , [x de donde: (x,xt2) (a-1,6) 1 x#2 = 8 + x=6 — — y > Luego, c=(6,8) . Si P(xi,y1) y PO=-AB > G-B = -(B-K) = K-23 6-x1 = 5 + xy=1 > (6-x1,8-yi) = (5,-1) 8-yi =-+-1 + y =9 . +» P(1,9) EjJemplo 8. Los vectores 2,6 y ceR?, cumplen que: 242620 y a-3b=2¢. Siendo 4 un vector unitario, hallar la nor- ma de bee. Solucién. De las ecuaciones dadas se tiene: Z = G-26 (1) , a = 20435 (2) Luego, G-26 = 20+3b > é = -56 Sustituyendo en (1) obtenemos: % =-48 + = 2a Entonces: $42 = #3 > Tfbrey] = 4) 181] Como 2 es un vector unitario + fal |=1 ee [1B+e] | = 4 Efemplo 9. En la figura adjurfta se tiene: Ne OM = 2x y OL=27/2 Si a=(2x*, dx244y?) y B= (duy?, - dry), hallar Solucién. Las componentes de 2 son OM y OL > B = (3x,24 » x20 2 Luego: 2(3x,22) = Fl2x*, dx2+4y?) . 2(gxy?, -4xy) = 2,3 _ 2 09 5x = Sx3 - + (5x,27) = (3x? Sxy?, gute 4y?+ Sxy) o x * 3* By 27 = duc? Sy + fy? Vectonres 19 13 = (xty)(x-y) (1) Si = (xty)? + (xty) = 2 (2) Sustituyendo (2) en (1) se tiene: 3(x-y) = 43 o. xny = 3 Efemplo 10. Sea el exdgono regular.de lado a, [7 mostrado en la figura. Al sumar BA, AC, DG y AE se obtiene un vector 33 hallar A \ D la norma de 3. Soluceén. Por geometria elemental sabemos que &£.=r=a y 2 =r/3, entonces: [JACI }=[|AE|f=a¥3 . por ser lados de un tridngulo equildtero. Trasladamos los vectores indicados a un sistema bidimensional con origen en A cu yo eje X siga la direcci6n de AD, y apli cando la ecuacién (5) tenemos: BA = ||BA]|](cos240°,Sen240°) = a(-3 - 5) AC = ||#E|1(Gos30°,sen30°) = a3 , 4) = a8, 4) DC = ||DC}|(cos120°,sen120°) = a(- 41,4) AB = ||HE]|(cos330°,Sen330°) = a3(Z ,-4) = ag ,-B) Luego, 3 = BA + AC + DC + AE = (28,0) ”. [IS[] = 2a Ejemplo ll. En la figura adjunta se tiene: + + Hlall=3, [[6ll=2llell=evT0 , Tga=1/3 y TgB=3. Hallar el valor de m de mo do que: > > > ma + 3b = ne Soluceén. Si Tga-1/3 + Sena=1/M10 y Cosa=3//70 TgB=3 + SenB=3/¥10 y Cos8=1/¥10 Un vector unitario en el sentido de a es (1,0) + 4=3(1,0) 2c Vectones 6 > c | 181] (-cose,-Sena) = 2710(-3//10,-1/YT0) = (-6,-2) [Vel |(Cos8,Sen8) = ¥T0(1/v70, 3/YT0) = (1,3) 3m - 18 =n (1) Entonces, si m(3,0) + 3(-6,2) = n(1,3) — ( 0 - 6 = 3n + n=-2 Sustituyendo en (1) obtenemos: m=16/3 Ejemplo 12. En el grafico se presenta una piramide regular cuyas aristas laterales miden 2a. Si el lado de la base 8B cuadrada mide a, calcular: [|F; + F2|]. Soéucéén. En el plano BVD se tiene: F, = BP + PV vV F, = DP + PV = -PD + PV = -BP + PV Luego: F, + F, = 2PvV > [{F, + Fo] = 21/PV1] * UP, + Pall = 2n = 2 A20)24(222 P de donde: ||Fi + F2]] = a/TZ | ESemplo 13. La figura adjunta es un tetrae V > > dro regular de arista a, M as v 4) ct ;unto medio de KC. Si Bavatvotvatvs, ha- llar la norma de 3. Solucién. En el ABVG: CB = 4, + ¥ En el AAVM: AM = %, + ¥s Efectuando la suma se tiene: 8 = CE + AM = GB + KC = MB + 13st = | {MB} ] Cattura de un tridngulo equildtero de lado a) ” Wal = 33 B Ejemplo 14. En el tridngule ABC, M es un punto de AC tal que AM = ZC YN Si la norma del vector BM es 2, hallar la norma del vector: ¥ = 2BA + 3BC. . ; —-. _. _ A M c Sotucién. En ol AAMB: BA=BN-AN = BM - 2uC En el ABMC: BC = BM + NC Vectonea 21 Luego: v = 2(BM - ANC) + 3(BM + MC), de donde: % = 5BM . Lei} = SEIBM]{ = 10 Efemplo 15. En la figura adjunta, el tridn- gulo OAB es isésceles con OA=AB y PH es perpendicular a OB y mide 6 unidades Si [JAQI1=2/10B]1, hallar ||PQ{1. Sofucéién. Sea OH=x + P(x,.6) AM OM AOMA = AOHP + 55 =a) +> $ < Luego: P(2,6) + PA = A-B = (2,8)-(2,6) = ( Ademds: AB = B-A = (4,0)-(2,8) = (2,-8) si [1AQ||=21)@B]| + AQ = $AB = $(2,-8) En la figura: PQ = PA + AQ = (=,2) + 2(2,-8) = $(11,-20) 2. WIPQIL = $ Aa1)2#(-20)? = Z 527 nj gular'de altura 3h y sus bases son tridngulos equildteros de lado 2h. P es punto medio de AB, Q es punto medio de FE ; hallar la norma de PQ. Sofucaén. Si por P trazamos PM||BC, entonces: A Cc P B — Ejemplo 16. La figura es un prisma rectan- a 2\ — 1so— LPR = AIlseIL =n Por el tecrema de Pitdgoras: ||PQ|}?= ||Pu! [7+] [MO] |? + | IPQ] |?= n?+(3h)? = 10h? + | [PQ]] = hv TO Ejemplo 17, En la figura adjunta, si P B(2, 10) ‘” es tal que el drea del tridn gulo APC es el doble del 4rea del tri4n- gulo CPB; hallar [|CP|]- A(-4, 2) <——_—_ . C(2,2) C2 A DL; eit aes, Vonenteal (x 4 y ) { x y {** = 2(10-y) > y=22/3 Entonces: CP = B-¢ = (0,28)- (2,2) = §(-3,8) Por consiguiente: ||CPI] = 5 /(-3)7+8? = 5 WTI B C Ejemplo 18. S81 ABCDEF es un exdgono regular cuyo lado mide a unidades, cal- 4 D cular el valor de: 115m + oF Il. F E Solucién. Trasladando los vectores a un sis tema cartesiano de origen A y aje y X sobre AD, tenemos: A x re = 0 ¥3 _1 se x AE = | |KE|{(C0e330°,Sen330°) = av3(*5, - 5) ir = & AE + AF = =(3.-¥3) oF = |G o °) = 2a(-1,- CF = | [CF] {(cos240°,Sen240°) = za( 3 4%) CF * CF = a(-1,-0/3) Luego: AE + 3CF = 2(3,-73) + Za(-1,-¥3) = 2(-1,-573) . (gk + StRIE = 8 4-1) 74(-579)? = 8 75 Ejemplo 19. En el rombo de diagonales D y d tal como se indica en la figure, hallar la norma del vector: vev,+ 7,473,448, aonde los vectores ¥y,.¥2.03 ¥ Vy llegan a los puntos medios de los lados del rom bo. Sotucién. Considerando un sistema cartesiano con sus ajes X¥ ae Y sobre las diagonales PR y SQ, respectivamente, tene- mos: ¥i= RR=F-R=(-2, 9) - 3,0) = (-40, 4 7 Po-0-F =. - (-3. 0) = (, 9) -G-B-G-(-7.-H- (, $) = (-2,-2a a+ dy . \ ~ Vectones 23 Yo2sk-0=G,-)- (0,9) = 2, -3a) Luego: Vet, +t, +33, +3, = (0,-4a) Ww & WwW DY . 70. 11. 12. HF} =a EJERCICIOS 2 : z : z > > . En los ejercicios del 1 al 5, si 2,6 y ec son vectores en R, demuestre le validez de cada afirmacion. 2+6+64+2 (Propiedad conmutetiva: A) 2 + (-2) = (-2) +B=6 (Inverso aditivo: As) Ssiatrb=0 + @=0-% > + , siatb-6 + 2-6 (Unicidad del idéntico editive) SizatbB=60 + 2=-F (Unicidad del inverso editivo) Mediente segmentos orientados demuestre la propiedad Az: ath = b+ Z, Dado él triaéngulo ABC, demostrer que: AB + BG + CA = 0. (Sug. User la def.3: AB=B-K) Dedos los vectores 2=(5,2), $=(-3, 4) y 0=(7,4)3 resolver le ecuacién: 2x +58 - 3b = 4c. Rp. x=(-3,9) Sea X un vector en R? tal que: (-5, 2)=2x+(1,-8). Si (-5,3)=txtr(2,-1), heller el valor de 2ttr. Rp. -2 Dedos los puntos A(5,1), B(-2,3), C(-3,-2) y D(1,-4); deter- minar el pimto X(x,y) de modo que: 3AB-XD = 3AX - 30D + EC. Rp. X(-2,197/2) Se tiene 2[(5,-1)+¢]=3(1,3)-(-1,2). Si A(2,3), B(3,-1) y el punto final del vector % en posicién ordineria, esté sobre €1 conjunto P={(x,y)/y=x?-1}; hallar les coordenedas de un punto P tal que: AP+2PG=AB., “Rp. P(-9,9) En e€1 exdgono regular ABCDEF, de lado a, F A hallar le norma de 3 sabiendo que: s = 4¢—p + Ips) 2 If 7 E 8 s 3 (AD + 5DE) + SEB. Rp. 38 15. 16. 17. 18. 19. 20. Vecto2res Siendéo 4=(5,-2), b=(2,-5) y ¢=(-3,1), hallar un vector uni tario en la direccién y sentido de V=2a-3bt 4c. Rp. a = (- 7 , 7) ay La base de la pirdmide regular de la fi “ENS gura es un exégono regular de lado a. A} XS as VsAeUReUA + : : : Le megnitud de rv es |[rv!J=[r]] [vif y su direccién es la misma > . - os que le de v, aunque su sentido puede ser opuesto, es decir, los > > vectores v y rv son peralelos. > . . . > Hota. Al vector rv se denomine méétiplo escabar de v. REPRESENTACION GRAFICA. Segtin que r sea positivo o negativo la gréfice de rv puede ser: r>0 r : Mot (rs)a = r(sa) Asociatividad My: la =a Neutro multiplicativo co Ny: ra = 6 +> r=0 0 a=8 Cero multiplicativo + > Mss -la = -a Inverso aditivo + + > + Me: r(a+b) = ra + rb Distribuidad respecto a la adicién de vectores. > > > (r+s)a = ra + sa Distribuidad respecto a la adicidén de escalares + > - Ho: |frall = Irf.] 2] | Magnitud respecto a mil tiplos escalares. 26 Demostraciédn de Mg: Vectonres i) Si reR y 2,beR?, tal que a=(x2,¥1) y b=(x2,y2)> denostrare- mos que: En efecto: r(atb) ii) Si r,seR y aeR*, ta > > En efecto: ra + ea wf ak 1 > ra 1.12 VECTORES PARALELOS m+ + L = + b) = ra + re ru r{(xisyi) + (x2,y2)] r(xitx2 » yitye) [r(xitxz2) » ryity2)] (rxitrxe . ryitry2) (rxi. ryi) + (rx2 , ryz) r(x: » yi) + rlx2, y2) ra + rb que a=(x1,y1) demostraremos que: > > + 6a = (rte)a r(xi,yi) + s(x1,y1) (rx, ryi) + (8x1, sy1) (rxa + sx, , ryi + sy) [(rts)x1 . (rts)yi] (rts)(x1 , yi) (rts)a ayt Dos veectores a y b, no nulos, son paralelos o proporciona- les ei y sdélo si uno de ellos es un miltiplo escalar del otro, es decir: al[B + Zs rb, Mrer wv) Observaciones: 1. Si vO y bf8 + & y rb tienen la misma direccién y sentido. a —— ~ >. Si r y rb tienen la misma direccién y sentidos opuestos. 6 b Ce ad a= rb a= rb TR SS. 1) lectco2res 27 <. Es conveniente establecer que el] vector nulo 9 es paralelo a todo vector, esto és: elj2 6 alfo . #zeR? > @ En efecto, si @||/2 + 6 = re = Oa (DeR) 3. Todo vector es perelelo ea si mismo. : > + trys a En efecto, si ieR + e@ = ta , por lo que: alle , FeeR? Ejemplo It. Determinar si los veectores dados son perelelos. 1) a=(4,-1) , b=(-12, 3) 2) a=(3,-6) , b=(1,2) Selucitn. 1) Si al[bB + (4,-1)=r(-12,3) F4e-24 > re-1/3 -1=3r —- r=-1/3 Como r es tnico y r (3,-6)=r(1,2) + {F257 7 F ~-6=r > yr=-3 Como r no es t:.ico + aN, es decir, no existe ningtn reR que eumple (3,-6)=r(1,2), pues esto implicarfa que 3=r=-3, lo que es imposible. Ejemplo 2. Demostrar que si @,5eR? son vectores pareleios y @#6 entonces existe un escelar r para el cual se giene: 2 = rb azr + Demostaaceén, En efecto, sean a=(xa,yi) ¥ b=(x2,y2)> y seen ai . : > : y G2 los gngulos de direccién de a y b respective mente. Segin las ecuaciones (4) se tiene: Sena, = —%3 , Cosa, = —*_ all ad] Sena, = —22 > Cosa, = Xa 131] sit Como por hipétesis 2 es paralelo a b, entonces: m(oi) = m(az) o may) = m(az) + 180° et V2 x1. %2 Si m(a,) = n(a,) + —%1 = = i Halt = EBL lat) UI oF _— ‘area. 2R Vectorzes >{lall belt de donde se deduce que: 1 7 Xz os Ya = ~z— y2 Het] } fol! > Por hipétesis 118]140, por lo que Tr es un nisero real r, 1oll entonces: X1 = FrX2 » Yai = TY2 r > Luego: (xisyi} = T(x20y2)s o sea: a= rb EjJemplo 3. Demostrar que si: allée. tlt y t4e + aft. Demostnacién, En efecto, si af@ y béa , entonces: i) aj {8 + &@ = rb /rer 41) BIS > B= se /seR Luego, a= rb= r(se) = (rs)¢ + alle Ejemplo 4. Demostrar que si a=b+¢ y Bt fas entonvgs: affa + Spe Demostazucién, (+) Supongamos que ata + srek/ d-rd Pero por hipétesis: B{|a > 3seR/ b=sa Luego, si Zsd.bera-sa=(r-s)a > ella (+) Anéloganente, supongamos que: @|{a + steR/ t=t2 Pero por hipétesis B/[& +» aseR/ besa Luego, si d=bré-satta=(stt)a + ala Ejemplo 5. Si a=(1-2n,1) y b=(-7,mt2), deterainar los valores de m, de moso que a sea paralelo e t. Sofucién. si al[B + arer/ 2 = rb + (1-2m,1) = r(-7,n42) {fares 4) 1=r(m+2) (2) Al dividir (1) entre {z e ) obtenemos: 2m*+3mn-9=0 : m=-3 6 n=3/2 Ejemplo 6 Si 4=(1,18) lo expresamos como a=%+¥, donde x1]B e fe. Si B=(-1,4) y @=(20,3m), hallar e1 vector i, Solucién, Si x||b > xX = rf-1,2) s(2m,3m) = sn(2,3) = t{2,3) a+ tet ‘oF + “+ x i Vectores 29 Luego, si a=xty + (1,18)=r(-1,4)4t(2,3) Te-rt2t (1) 18-45 3t (2) Resclviendo las ecuaciones (1) y (2) obtenemos: r=3 y t=2 J. & = (-3,12) Ejemplo 7, Se tiene que: ea=(m,2m), a-b=(2n,p), bl Ja y la norma de 4-6 es 20. Hallar la norma de 6. Sofuciéns Si bl]2 + B= ra = r(m,2m) = mr(1,2) (1) a-6 = (2m,r) + (m,2m)-mr(1,2) = (2m,p) > (m-nr,2m~2) = (2m,p) Por igualdad de vectores: m-rm = 2m , de donde: r=~1 Luego, en (1): 6 = -m(1,2) + [[BI| = (2) Ademés: 8-6 = (m,2m)+m(1,2) = 2m(1,2) + |la-bll = 2n/% si |[2-Bl{=20 + 2m/3 = 20 + m=2/5 . Finalmente en (2): bl l= 10 Ejemplo 8 El vector a=(3,0) se descompone en dos vectores 6 y @ paralelos a los vectores (2r,- 3r) y (p.-3p) res pectivamente, donde r#0 y p#0. Hallar la longitud de By @. Solucién, Si bi |(2r,-3r) + 6 = £(4,-3) = s(4,-3) el(p.-3p) + @ = p(1,-3) sid =6+% + (3,0) = 5(4,-3)tp(1,-3) «+ 4 3°484P O=-3s-3p Resolviendo el sistema obtenemos: s=1 y p=-1 Luego: 6 = (4,-3) + JBI] = 4(4)?4(-3)? = 5 @ = -(1-3) = (-143) + [Vell = “(-1)24(3)? = 770 Ejempio 9. Dados los vectores a=(2a,2), b=(6,n), @=(c,3n). Si al[B[[%, cealculer e1 valor de ante. Soflucéén, Si al [|b > Beth + " (2a,2) = t(6,n) 2a = 6t 2= tn Eliminando t del sistema obtenemos: an=6 si B]1t + Bere + (6,n)=r(c,3n) «> (6=re) (n=3rn) de donde: r=1/3 y c=18 Por tanto: ante = 24 Pecicrzes Be(V5,-/20) y c=(/12,75); heller [Fit]. {Fel i, iendo sit, Valle y ¥aitve=(-7,4). Ejemplo 10. 3i 1 1 Sokucién. Si Vif |b > > Vv Yi = s(¥5,-2/5) = s¥5(1,-2) = t(1,-2) 2ll@ + V2 = k(2v/3,73) = &v3(2,1) = r(2,1) Entonees, si: t(1,-2) + r(1,2) = (-7,4) — ftter = -7 -a2ttr = 4 Aesolviendo el sistema obtenemos: r=-3 y t=-2 Luego: ¥ = -2(1,-2) > |f¥all = [-21/(1)?4+(-2)? = 2% ¥. = -3(2,1) + |l¥el] = 34(2)?#(1)? = 375 “Wall. lel = 30 Ejemplo 11. La figure eadjunta es un octeedro regular de aristea ea en donde ac~ > + Vi > > > > tuan los vectores vy , V2» Va» Vy y vs. He- > . > > > > +> + llar |[s]] si, = = Vi + Vo + Va + Vy t V5. ods U3 . > > b Sofuciéa, Los vectores vi y v3 son psralelos Ve J = - > > y de sentido opuesto + vi = -¥3 = + > + > — — Ademés: OA = viut¥s * 8 = Vz + OA = AB TIST] = TLABEI a Ejemplo 12. En la figura se tiene un exaégono reguler cuyo ledo mide eae. Si HIF Al=UPot=dlFsll=ilFat l=] Bs] f= a, hallar LiSl], donde: 3 = Fy + F, +P, + Fy, ¢ Fs. Soluciéa, Fi-F, ¥ Fo=F, por sér paralelos y x Ge le misma megnitud, direccién y *5§ s-utido. Entonces: § = 2F, + 2F. + fF, Trasladando estcs vectores a un sisteme de ejes rectengulares se tiene: a(Cosg0°, Sen90°) = a(0,7) ‘th ” ul . fi fe a(Cus6L®, send”) a(S ’ 4) 4 w Mt a(Cos180°,Sen180°) = a(-1,0) luego, 5 = 2a(0,1)+e(1,/3)4a(-1,0) = a(0,2173) + 113] [=e (2473) Vectonres 31 EJERCICIOS Demostrar que: aljfe, BI|/t y t4e > Bf fb > > Sezcestrar que para vectores no nulos 4, a1 » $ ’ ty : alla: . B]Ib: y allB + B,t [bs Semostrar que si a y 6 tienen la misma direccién entonces: > > Ha + SH = [fell + 1ét Si €=(2, 2n-3) y t=(1-n,-5), determinar los valores dem de modo que @ sea paralelo a b. Rp. m=-1 6 m=7/2 $i a=(m,5)+(3,3), be4(-m,-3)-2(1,2) y al|bs determinar el va lor de mr. Rp. m=2 Tadcs los vectores a=(a, 3m) y $=(-2m,b). Heller atb de modo gue etb=(8,-4) y sea al }t. Rp. 5 Sean los vectores a y 6: a=(a,2a), a-b=(2a,p), 6/[& y la nor ra cs 2-6 es /T12. Hellar ||B{]. Rp. 2/7 P El vester a=(x,y) es paralelo al vector b=(2, 4), tal que: i = (—~ , -L) es un vector unitario paralelo a ambos. Hallar Ye V5 > > el vector a. Rp. a=(+1,22) Sean a y 6 dos vectores en R?, tales que 6 es el inverso adi ivo de a. Si 6 tiene el mismo sentido que el vector c=(-1/3,1/4) y [fall=5, hallar x=2+2b. Rp. x=(-4,3) fallar la norma de la suma de los vectores unitarios u y v ’ si ulfa y V1I{b sebiendo que a=(4,-3) y b=(-5,0). Rp. V10/5 Los vectores a y 6 son tales que & es del mismo sentido que > z a 1 db, = ( , +) y b2(1,3). watlar 2x- y Rp. 1 > Hall va” va K En la figure adjunta tenemos un cubo y como “techo"” una pirdmide regular, todos de aris AX> G taa. Sis = DE + BA + KC + HC + FG, heller B la norma de 3. Rp. a ' Je -T Ax 32 13. 1h. 15. 16. 7. 18. 19. Veetoses El vector c=(2,-1) es expresado coso t=atb, donde los vecto- res a y B son paralelos 4 x=(3e,4m) y=(-3n,-n), respectiva mente, siendo mfO y nfO. Hallar a-b. Rp. -3(48, 31) En la figura adjunta, sea 0 la inter- seccidén de las diagonalee de un cua- @rado ABCD. 82 0 ss cl baricentro del tridngulo tedsceles APD con | [AP] |= [IPD] }. Heller RQ. Rp. WQ=(1/2,-3/2) Dedos los vértices consecutivos de un paralelogramo A(7,-1), B(-3,1) y C(-5,5). Determinar el cuerto vértice D y la longi tud de la diagonal BD. Rp. D(5,3) , 2v17 La figura sostrada es un paralelograno rectanguler donde |{{ABl{=4a, | |AF][=3a |{AG} 1=60. Haller |{3l} si: 3 = 3D + AG + AB + AF Rp. 13a Si a=(a,b) y B=(1/2,-4/3) son dos vectores en R?. Hallar atb > si |/a}|=(1/3)/73 y si a y 6 tienen sentidos opuestos. Rp. 5/3 En ln figura ABCD es un cuadredo de B c lado 3a y A'B'C'D' es un cuadrado ds lado a, si |]D™D[|2a hallar BrQ. —_ no. pen eg ly. 4 pe Oe = glae-a) La figura representa un prisaa super puesto a un cubo, si todas las aris- tas son de longitud a y si: 3 = EC + OB + 3oK + am + tec Hallar el valor de ||5][?- Rp. (12 + /3)a? Vectores 33 1.13 PRODUCTO ESCALAR DE VECTORES Dados los vectores a=(ai,a2) y b=(bi, bz), el producte esca lar o interno de a ¥v 4 se denota por a.b, y se define pors a.b = (a1,a2).(biyb2) = aiba + aabz (10) Observaciones: i) El producto esealar de vectores es una operacién cuyo resul- tado es un escalar y no un vector. 4i) Si 1,Ber™, entonces: n 4.8 = arbi + azb2 + «2.25 + a,b, = 221s PROPIEDADES DEL PRODUCTO ESCALAR. Si a,b y © son vectores en R? y reR es un escalar, entonces se cumplen las siguientes propiedades: Ei: 4.6 = bea Conmutatividad E2: r(a.b) = (ra).6 Asociatividad escalar Es: 6. (ate) = t.a + 2.6 (348) .Z _ 2 ’ *.2 Distribuidad Ey: asa = [Jal |?20 Magnitud respecto al producto escal. Es: aa=0 + Bo =@6 “$7 INTERPRETACION GEOMETRICA DEL PRODUCTO ESCALAR EN R? Sean & y &B dos vectores y a-b (el vector que va de Ba A}. Sia es perpendicular a 6, ocurre que la representacién geométrica de los vectores a,b y a-B es un triaéngulo rectangulo, para los cua- les, por aplicacién del teorema de Pitégoras se tiene que: ya-B)]? = Wal]? + TBI? A (a-6).(a-6) = [Jal]? + [BI1? (Ey) +3 + aa - a.b - 6.2 + 6.6 = [fall7+1/BI]? (fe) + ffap[?- 20.8 + JBI]? = Plerl2+ibll? (By) + de donde: -2a.6 = 0 + a.6 = 0 8 Como hemos establecido la condicién de perpen or dicularidad para a y %, entonces podemos dar 34 Vectones la siguiente definicién. 1.14 VECTORES ORTOGONALES Dos vectores a y 6 son ortogonales si y sélo si a.b=0. Si es el caso que a y % som ambos no nulos, entonces se dice que los vectores son perpendiculares y sanotaremos: aibB + 3.8 = 0 (11) Por ejemplo, si &=(1/2,-3) y B=(-2,-1/3), entonces segin (10): a.b = (1/2)(-2) + (-3)(-1/3) = -1 +1 = 0 Como a y % no son nulos, entonces: 416. DEFINICION 6. Para cada vector a=(ai,a2)eR?2, definimos un co- rrespondiente vector a*eR?, que se lee ontogonal a 2, mediante: a+ = (-a2,a1) (33) > = Graficamente el vector 4+ se obtiene > haciendo rotar el vector a, sobre su punto inicial, un dngulo de 90° en dj receién contraria a las agujas del re loj. ane > + Se verifica luego que si a14a+, enton >~ + ces a.a'=0, -~ + En efecto, a.at a (ai,az).(-a2,a)) = -8182 + agai = 0 > + .» atat PROPOSICION 1.2 (Desigualdad de Cauchy-Schwartz) Sean i y B vee tores en R*, entonces se cumple: 4) 18-8) ¢ HIT BHT id [BBE = [al] PBL] ayy Demostnacién. 1) Si =e & b=8, entonces se note clarsmente que la proposicién es vdlida. + Supongamos que a#8 y b#@ y consideremos la funcién para un nime- ro reR: f(r) = [fatrb]|? = (Zerb). (Serb) (1) y ocurre que f(r) > 0 , ¥reR Vectones 35 Desarrollando (1) nos d& el polinowio de segundo grado: f(r) = (b.b)r 22(4.b)r + (8.8) Completando cuadrados se tiene: - (bb) lee 4 208.3) GP _ (2.3)? 5 3 f(r) = ( yr + (EB) r+ (8): (6.8) + (a.a) - (8.8) (+o BB)* Gi6.0-G.0) (B.B) (2 + 5)" Bb Si hacemos r, =- } + f(rQ) = GOO.) ee (2) Como f(rg) 20 y %.% = I1B112>0, esto implica que: (4.8) (6.6)-(8.6)220 ++ (8.6)? < (4.a)(6.5) +> fa.Bl? < (lar? tialy? atl < (al) Ell ii) Demostraremos que: ja.8] = fal] (lel! +7 alls () sa GiB > 12.8 = URTE TIBI En efecto, si al|t + G=rb Luego: [4.6] = |(rb).B] = [r(b.6)! = Ir f] 1d] 1? = [rl lBUlbr = [feb] tt) la.b] = [lal HSI (+) sa fa.B] = Plarlifbil + aft En efecto, si [a.b] = [fall/]Bi| > (&.6)?=]!at 121 1bL]? + (a.6)?=(4.2) (8.6) Sustituyendo en-(2) ocurre que: £(r,)=[atrgb|=0 * » Barb -a- bbe + be xh $.6 Por tanto: al |t PROPOSICION 1.3 (Desigualdad triangular). Sean a y 6 vectores + . > en R*, entonces: [laré} | < blali+tibel Més atin: |la+Bll=}/EE}+bIBT] si y sélo si wm vector es un mflti plo escalar no negativo del otro. Demosinacién. En efecto: | fat+d]|2=(a+b). (at) = ffalf2+ 2a.6 + Jb] ]2 36 Vectonres + | (a+b) ]? < Pall? + 248.8] + [1B]? Por la desigualdad de Schwartz, se tiene que: > [fab] |? < Pall? + 2u0alt Elly + 1181? < ((all + [tp 2 [a+b] < [all + Ube * Ejemplo 1. Demostrar que: {JasBl 2 = Papp? + }{B) [2 + 22.8 (atb). (a+b) (E.) a.(atb) + B. (+b) (E3) Demostnacién. En efecto: | {atbl]? = B.a + ab + B6.8 + BB (Es) = a.a + 6.6 + 28.6 (E: y Ea) . Ufaabl 1? = PPal]? + EBL]? + 28.3% (£,) Ejemplo 2. Demostrar que a+b y a-b son ortogonales si y sdlo si Hatl=Hell. Demosinacéén, Demosttraremos primero la ortogonalidad. En efecto, por hipdétesis: Hrall=H1Bil o> flary? = poppe > [lal]? ~ [Bl]? = 0 + (a+b). (4-5) = 0 Por tanto, segin (11), atb y 4-6 son ortogonales. Ahora demostraremos la igualdad de las magnitudes. En efecto, por hipdétesis, atb y a-6 son ortogonales + (a+b). (2-8) = 0 > 2.8 - 6.64 8.2 - BB =0 > Tall? - HBIe = 0 > PPaly2 = BE? Por tanto: biadd=!1bt] Ejemplo 3. Demostrar que: (a+b) + = at+ $+ Demostnacién. En efecto, sean: a=(ai,a2) y b=(b1, ba) > &+ 6 = (aitbdi,a2tbe) > (8 + B)* = (-a2-b2,a14b2) Vecicaes 37 = (-a2,a1) + (-b2,bi) “ (2+ b)4= a+ + B+ Ejemplo 4. Demostrar que si el vector ¥=(b4+.0)8-(24.3)6 es para ‘ lelo al vector ¢. Demostnaczén. En efecto, sean #=(82,82), b=(by,b2) y G=(e1,¢2) vectores en R?. v (B*.0)a = [(-ba,b1).(c1,¢2)) (21,42) (-b2¢1+b1¢2)(a1,82) (-arbeei + aibjyea » -az2b2e1 + a2bic2) (1) (a*.¢)B = [(-a2,a1).(c1,¢2)] (br. ba) (-azeataicz) (by, bz) (-a2bici + @ibyeg , -azbdec, + a1b202) (2) Restando (1)-(2) obtenemos: v (a2bic1-ay bec) azb,¢2-81b2C2) [(azbi-aibs)ex. . (azbi-aibz)c2] = (azbi-a1b2)(e,,¢2) El coeficiente de ¢ es un escalar, por tanto: > > > > v=.re + vile H Vv Ejemplo 5, Demostrar por métodos vectoriales, que un tridngulo inscrito en un semicirculo es un tridngulo rectangu- lo. Demostnacién. Supongamos el ABCA inscrito en el semicfreulo cuyo cen- tro es el origen y cuyo radio es [/b/|. Segin la figura debemos probar que BCLCA. En efecto, BC.CA = (b-a).(bta) $.6 + ba - 3.6 - 2.8 1 17-1tall? Pero [/é] |=! al] per ser radios del semicfirculo. Por tanto: BC.CA = 0 + BCILCA Ejemplo 6. Resolver la ecuacién: 2[(1/2, 6) + Tte | = jt - 2xt si t=(1,0) y F=(0,1). . Solucéén. 2{(1/2,6) + (1,0)* - (xaex2)] = (0,4) - 20xa,x2)" 38 Vectores (1,2) + (0,2) - 2(x1,X2) = (-1,0) - 2(-x2,x4) (2,14) = 2(x1,x2) - 2(-x2,x1) [is toe + (1,7) = (xptxes%2-x2) (7 = x2 - Xi de donde obtenemos: x1=-3 y x2=4 + x = (-3,4) Ejemplo 7. Sean a. beR?, demostrar que si 2at-6 = 2b+-a, enton- ces at es ortogonal a a-b. Demostracién. En efecto, si 24°-b=26'-% + 3-6 = 2(b*-8') (4) Aplicando el ortogonal a ambos miembros de (1) y haciendo uso de las propiedades: (a+b)* = $*4+6+ (a*)* = -8 se tiene: (4-5) * = 2(b*-a7)* > e481 = 2(-b + 3) + 4(a-B) = 2(8+-8+) (2) Sumando (1) y (2) obtenemos: 5(a-b)=6 > 43-be6 Luego, (atb).(a-b) = (4tb).0 = 0 Por tanto, segtn (11): (a+b) 1 (a-B) Ejemplo 8 Hallar la norma del vector b=(-3n,m), sabiendo que ha sido descompuesto en el vector a=(-5, 3) y en otro vector paralelo al vector ¢=(1,1). Solucién, Si b-m(-3,1) + [IB]] = |m|/(-3)24(4)? = [mlVIO (4) y si: beatreé + m(-3,1) = (-5,3)4r(1,1) Multiplicando cada extremo, escalarmente por (7,1)", se tiene: m(-3,1).{-1,1) = (-5,3).(-1,4) 4+ r(1,1).(-1,1) > m(3+1) = (543) + r(0) , de donde: m=2 Por tanto, en (1) se tiene: 181] = 2/10 a tne a. 7 + * : rgsempto 7. Si @ y 6 son vectores unitarios y paralelos, haller +, > la norma de attb. Soéucién. Sabemos que sir alld > a= rb o bien: allt > |at.6=0 Entonces: | lat+B] 12 = flat} |*+284.6 + [Vet t? (1) + 2(0) # (1) . [farteBl] = v2 Vectones 39 Ejemplo 10. Si a=(-6,15), b=(-2,9) y G=(-2m,3m) y se sabe que: xty=a, x[ [Be y[l¢. Hallar x.y4 t(-2,9) (1) Sobucién. Si x[ [6 + & tb > é sm(-2,3) = r(-2,3) (2) x + y > x > = sec > y yile + Luego, siz t(-2,9)+r(-2,3) = (-6,15) + {72t-2re-6 * ters3 9tt3r=15 > 3ttr=5 Resolviendo el sistema obtenemos: t=1 y r=2 Sustituyendo en (1) y (2): x=(-2,9) , y=(-4,6) . >? Ww. keyt = (-2,9).(-6,-4) = 12-36 = -24 Ejfemplo Il. Si a, % y atb son vectores unitarios, hallar la nor ma del vector a-b. Sofucéén. Si el vector atb es unitario + | |2+B| J =1 > | Jaeb] | ®=1 > | /af [242a.B+ | (Bp pt=4 > 44 28.64 121 ++ 2.6 = -1/2 Luego: |la-bJ|? = |Jay{? - 22.6 + [JB] {2 = 1-2(-41/2)41 = 3 . [[a-BI] = v3 Ejemplo 12. ‘si atbee=0 y [[2][=2, | [bl [=5, [[e]]=8s nallar 4.6 Sotucién. Sj atbee=-@ + ath = -G > [a+b] |? = | [et f? + [fal f?+2a.be[ [bi l? = | fel {2 > 44 24.64 25 = 64 de donde: a.b = 35/2 Efemplo 13, Si 3=(1,x), $= (2x, x) y =(2x,;-1), donde x es un nié- mero real; hallar Ja suma de los elementos del con- junto M = {(x,y)/(a-c).b = a.c-1}. Sofucéén. Tenemos: a-c = (1,x)-(2x,-1) = (1-2x,x+1) +M = {(x,y)/(1-2x,x+1).(2x,x) = (1,x). (2x,-1)-7} = ((x,y)/2x-Ax?4x74+x = 2x-x-1} { (x,y) /3x?-2x-1=0} Por tanto, si M = {xa,x2,} + x )tx,=2/3 40 Vecionres ’ >, + Ejemplo ys. Dado el vector b=(2,3) y la funcién f:R?+R/f(p)=p.6 > El vector 4 es tal que f(a4)=-16 y aj[@=(1,2). Caleu lar la norma de a. Solucién. Si t(p)=p.6 + f(a) = a.6 = -16 al{@ + a = re = r(1,2) (1) Entonces: a.b = r(1,2).(2,3) «+ -16 = r(2#6) + r=-2 uego, en (1): a = -2(1,2) > [Pall = |-2|/T#7 = 23 Ejemplo 15. Sea el cuadrildtero PQRS. 5 2 a = + — R Sean: a=PQ , t=9R » c=RS y * ad-5P. Wallar ¢.d si se sabe que: d 4 Hasbli=7, Hell=3 y [ldll=5. P a Q Soflucién. De la figura obtenemos: a= atote + [pd-e] [=] [a+b] ]=7 Elevando al cuadrado: ]}a}J2-2d. G4} 8] ]2= 49 +25 - 2.4 + 9 = 49 de donde: t.d = -7.5 Ejemlo 16. En la figura A,C yEson *F 4 B puntos correspondientes a vértices de un tridngulo equilétero ins crito y los segmentos AB , CD y EF son tangentes a la circunferencia tales que E c ]aB1 |= 3» [1eDI I-24 , EF] I~ 5. Hallar + -u, si 3 = AB+CD+EF y u = (2,273). D Sofucién. Traslados los segmentos AB. CD y EF sobre un sistema car Y tesiano de modo que sus puntos inicia- 5 les céincidan con el origen. Entonces: es 3 AB = |JAB[|(Cos0°,Sen0°) = 3(1,0) eof px EF = 115] essay? »Sen120°) = 5-44) 4 EB = | 65] |(cos240°,Sen240°) = «(-4,-Q) D Luegor 8 = (3,0) + (- 3,903) + (-2,-v9) = ¢2,%) ~H#3 = Por consiguiente: 3.u = 3(-3,73).2(1,73) Vectones 41 Ejemplo 17. En la figura, m(f¢ABC)=90° y [JOB] [=3 . HalYar x si: x = 08.06 + OA.OB - OA.00 Sofucién. x = OB. (OB+BC)+0A.OB-OA. (OB+BC + x = | JOB] ]2+0B.BC+OA.OB-0A.OB-02. BC = |[OB[]?+BC(OB-OA) = | |OB[|]?+BC.AB Pero: BOC.LAB + BC.AB = 0 . x = | [OBI]? = (3)? = 9 Ejemplo 18. Dados a=(m,3p) y t=(-2p,n). Hallar el valor de — de modo que: a+b=(8,-4) y a.b=0 . n-2p=8 + m=2pt8 (1) 3ptn=-4 > n==3p-4 (2) Adenés: (m,3p).(-n,-2p)=0 + -mn-6p?=0 + mn = -6p? (3) Sustituyendo (1) y (2) en (3) se tiene: (2p+8)(-3p-4)=-6p? de donde: p=-1 , luego, en (1) y (2) obtenemos: m=6 y n=-1 Sotuciéa, Si (m,3p)+(-2p,n)=(8,-4) > { Ejemplo Jf. Un tridéngulo DEF se encuentra sobre un plano inclinado como se muestra en la figura adjunta. Hallar el vector DF. Solucién. Tenemos: DF = DE + EF HIOA]] = %(12)24+(5)? = 73 Un vector unitario en el sentido de OA es: Entonces: = 30 = (8,43) ; EF = 2ut = 2(--73.45) = 39.34) . 36 15 1g Zhy _ oe = (84%) + 61.24 = (2,9) Ejemplo 28. Dados tres vectores unitarios 4 ’ b y @ que satiafsa > + - cen la condicién atbte=6, ealcular el valor de: > Bb + Bt + Bee Solucién, Si atbte=@ + Ath = -3 + []aed}] = 1-2] 42 Vectonres Elevando al cuadrado ambos miembros se tiene: [op [2 + 28.6 + EPSP]? = PEP]? «> 1428.be1=-1 + 2.6 = -1/2 amente se ahtiene: b.t = -1/2, Bee = «1/2 “. 2.6 + 6.3 4+ 2.8 = -3/2 Ejemplo Ji. En la figura adjunta, los triangulos OCB, PBS y RST son todos ellos semejantes. Hallar ‘RT si P y R son puntos medios de OB y PS respectivamente. Sofucién. La figura muestra tres trian gulos recténgulos isésceles, en donde: [|OB||=4/2 y ||PS||= “(2/2)?4(2/2)? = 4 = 44) _ ¥ 5(4,1) Un vector unitario en el sentido de OB es: u = = 42 Entonces: PB = 2/2u = 2(1,1) ; BS = 2/2u+ = 2{-1,1) = (-2,2) 1k = PB + BS = (2,2) + (-2,2) = (0,4) ~ ol we XN Luego: Un vector unitario en el sentido de PS es: ¥ = {04 4) = (0,1) Entonces: RS = 2¥ = (0,2) y ST = 2y4 = (-2,0) -. RE = RS + 8T = (+2,2) Ejemplo,72. Sea ABCD un recténgulo, una de cuyas diagonales tie ne por extremos A=(-6,1) y C=(-2,8). Si los lados de mayor longitud tienen el mismo sentido del vector B=(2,1)3 ha llar los vértices B y D. c _ _. y Soluciéna. AC = C-A = (-2,8)-(-6,1)=(4,7) \i ce: KRII> 2. TR e vlan ay vib my ya * AD = Pies i) D BC} a + BC = t(-1,2) B Como AG = AB + BC ? + (7) = r(2,1) + t(-1,2) ~ De donde obtenemos: r=3 y t=2 0 * Por tanto: AB=3(2,1)=(6,3) + K+kp (6,3)+(-6,1) = (0,4) BC AD K+ap . (-6,1)+(-2,4) = (-8,5) bol Ml v| " C = AD = 2(-1,2) = (-2,4) + 10. Vectonres 43 EJERCICIOS Sean & y % vectores en R?, Utilizando las propiedades del producto escalar, demosbrar: a) [lal l? - [1a-B11? b) | [arbi l? + [fa-BE? 42.6 2c lal]?+] 16117) > . Demostrar que los vectores 4 y 6 en R? son ortogonales, si y s6lo si: + base]? = |lalt? + Bel? Dados los vectores a y %, demostrar que: a) (24)4 = -3 ec) 24.4 = 2.6 b) a+.8 = -2.6+ a) [1/241] = 18] Dados los vectores a y %, demostrer que: a) a.b = -| [all HII — 2 y t tienen sentidos opuestos b) [fa+bl] = [lal] + TIBI] + @ y 6 tienen e1 mismo sentido Deducir de la desigualdad triangular que si a y % estén en R?, entonces: [Hal t-HBI| < WeeBit s Mate L EL (Sug. Escribir: a=b-(2-6), y aplicar la Proposicién 1.3) + Demostrar que si a y % son vectores paralelos en R?, enton- ces: Ja.b) = Tall 1HBll Si a y % son vectores en R?, demostrar que: a) la.b¢] < Hall Tei b) [a4] = 121] [Bt] + 24% Demostrar mediante un contrejemplo que a.b=6.6 no implica ni que b-<, ni que a=o. Siendo 4=(2,-3), b=(-2,1) y t=(3,2), hallar un vector aut rio ortogonal al vector v=5a-3(bte). Rp. u= (3B, +35) Si a=( 4m, n- 3) y b=2 2,m+3), determinar los valores de m tales que a sea perpendicular a b. Rp. m=1 6 m=-9 44 71. 2. 13. 14. 15. 16. 17. 18. 19. 20. 21. 22, Vectonres Expresar en la forma vextty} el vector cuya longitud es 3/75 y es ortogonal al vector w=2a-3bt5e, siendo a=(-1,2), > b=(3,-5), ¢=(3,-4). Rp. ¥=31-6] 6 ¥=-3i+6j Sean los. vectores a=(m?-3,m-1), 6=(4/m?,4/m), donde mf#0 es un nimero real positivo. Si a y % son ortogonales, hallar el vector v=9b-4a. Rp. v=(19,22) si f=(1,0) y J=(0,1) resolver para x: a) 3[f++%-(4/3,2)] = (9,-11) 42% 53 Rp. X=(5,~1) b) (6,12)+3[(-2,1/3)-2}+324+%] = 4ke2jt-x4 orp. X=(~1,-5) c) 3(-2,~3)* + g[k+t*-(3,-1)]* = (5,2)+-2%+ . %=(5,4) > > Sean ay % dos vectores en R?. Si 2 es unitario y se cumple > > + > > que 4.6=9/4 y a.(6+9)=3, hallar &. Rp. a=(2v7/4, 3/4) Seen los vectores a=(x.xt4), b=(5x-5.x-4). Si x>0 y 2.b=-10, hallar |/a+b]]. Rp. 5 Sean los vectores a, $ y c tales que: [1a] [=”23, [1] J=3”2 y $.t=12. Si 2=6-2, nallar | [3]. Rp. 472 Sean los vectores a, By © tales que: a=bte, }a[]=5. |b fs 2/5 y b.c=10. Hallar |J@l|. Rp. 5 Si a=(2,x), b= (x,-2x) y G={x-2,x+1), donde x>0 y si (ath). t= a. $t1, hallar el vector 3= atte. Rp. ¥=(5,1) Si atb-c=e y [[Al[=2, [|bll=4/3, | !Sll=8; calcular 2.2 Rp. 19 Sea el rectangulo ABCD de area 48u? y cuyes dos vértices con secutivos son A=(-2,5) y B=(2,1). Si la diagonal AC tiene el mismo sentido del vector ¥=(5,1), hallar los vértices C y D. Rp. C=(8,7), D=(£,11) . > Si aeR y u=(a-2,5-3a) es un vector unitario, hallar el valcr de: [Ja(u+2u2)+204]]. Rp. 5 6 210 Sean @,beR?, ambos unitarios, demestrar que: liga + Sol <4 loeeierze Zé RELACIONES ENTRE VECTORES 1.15 ANGULG FORMADO POR DOS VECTORES Sean a y b dos vectores no nulos gue tienen el misnm ori- gen y sea 6 el menor de los érgulos pesitivos fornado per dichos vectores, oue satisface: O< €<¢ T. Los vectores 2, b y la diferencia a-6 forzan un triangulo cuvos lados miden llall, TIBI] y [la-BI]. (Figura 10) Por la ley de los cosenos se tiene: }ya-Bl =] al )?44 181 |27-2] Jatt | [5] [cose Desarrollando el] cuadréco del prirer mienbro obtenemos: Figura 10 []a-B] |=] fad y2+t |b] [2-22.68 Comparando ambas ecuaciones se deduce que: [lal] |1Bi|coss (2) de donde: x3 Cose = —S*°____ (13) Halli flbll Ejemplo 1. Hallar el valor del Angulo que forma el vector z que va de A(4,5) a B(6,4), con el vector & que va de C(-3,1) a D(-2,-2). Solucién. & = BB = (6,4)-(4,5) = (2,-1) + []A]] = 7% > _— a aa as satae — b= Cb = (-2,-2}-(-3,1) = (1,-3) > Joi] = “70 Luego, segin (13): Cose - (2:71). 01,73) 2 243 2 1 (¥5) (10) 2 v2 2 8 = 45° Ejemplo 2. Hallar la nerna del vector a, sabiendo que a y 6 for man un éngulo Ge 60°, d=a+b, |Jalf=3 y || Bl [=5. Sofucién. Si d=a+b + |JJa]| = [[e+bT| 16 Vectenes Flevardo al se tiene: [Ja]|? = [JZ] ]2+22.5 + JIB]? 0,7 )% fegin la ecuacién (12): [la] ]2= Elall?@+2t [all 1S] [coséo°+| 1B] |? = 9 + 2(3)(5)(1/2) + 25 = 49 ’. {fal} = 7 Efempl1o 3. Calcular a. donde z y % son ¥ vectores de la figura adjunta para los cuales: [[aji=4 y [|b] ]=23. wy Solucién, Si € es el Angulo que forman ambos vectores, entonces: 6 = 90°-(12°+18°) = 60° Lueg>, segdn (12): 4.56 = Lal] 1]B[ cose a.b = WS “| (4) (273) Cos60° Ejemplo 4. Los vectores a y 0 .orman un Angulo de n/6 redianes. Sabiendo que |/a]|=/3 y [[é]|=1, haller el angulo q! > > > > fornan los vectores u=2tb y vea-b, Sofuceén. Segin la ecuacién (12) tenemos: Halll lBllcos(s/6) = (73) (1)(/3/2)= 3/2 LiSLIELT {cose +» (3+6). (4-6) = [12+t11 18-8} [cose > [lapl?-]1Bly? (| [al |?+28.b+] 18] ]?) (| [2] 4-22. b+ | [BL {2 )Co8e + (73)?-(1)? = (A (73) 242(3/2)+(1)2) (AVS) ?-2(3/2)4 (4) 2) Cosa ae dcnde: Cos@ = 2//7 + 6 = arcCos(2//7) . > + + : Ejemplo 5. 98 vectores a, % y c forman dos a dos un angulo de 60°, sabiendo que | {fal{=4, | }B]f=2 [Je] {=6, deter- minar el wédulo del vector d=atbte, Solucién. Si Vaa+b+e + [[Sf] = |fasbesyy Eleveando al cuadrado se tiene: HIVE]2= [PEL | e+] PB] P24] [Sf 2428. B28. 3423, 2 = Pall] Oise lepsesoc yar CSHeearieiselsHiBepraep Cos60°, Vectores 47 J[VPP2= 1644436 + 2(4x2 + 4x6 4 2x6)(1/2) = 100 “. TIVE] = 10 Ejemplo 6. Los vectores a y 6 tienen igual longitud y forman un dngulo de 60°. Si 1a longitud de ath es 4 unidades mayor que la longitud de uno de ellos, hallar la longitud de a. Solucién. Tenemos: a6 = ]Jal|{ [6] |cos60° + 28.6 = | [al] laeBl] = 4+ Tal! Elevando al cuadrado: |[a||?+2a.6+[{B[{? = 16+8[|al]+] a] [? Como |fall=I]6]] * [lal]?-4)lall-8-0 ++ [fal] = 2 + ¥7+8 e. J/a] | = 24273 Ejemplo 7. Si el vector a=(-78,/50) gira 45° en el sentido hora rio se determina el vector b=(x,y)- Hallar x+y. Sotucién. Si | |b ]=]laf] > ¥x7ty? = V8+50 > xt4ty2 = 58 (1) > cosyse = —— ash, YZ. (-2¥2, 572). (x,y) Wane =|? (58) (¥58) de donde: 2x-S5y+29=0 + y = $(2x429) (2) Sustituyendo (2) en (1) obtenemos: x?+4x-2150 + x=-7 6 x=3 Elegimos x=3 por cuanto el lado terminal de & est& en el priner cuadrante. Luego, en (2) se tiene: y=7 .. xty = 10 Ejemplo 8 Los vectores a y % forman entre si un angulo de 45° y la norma de a es YZ8, Hallar {[6]], sebiendo que a-6 es perpendicular al vector a. Solucién. Si (a-b)ia » (A-b).4 = 0 + %a-2.6=0 + [[al{?2 = 4.6 Tall? = TTP LLB] {cos30° af3 = | PbE] 3/2) * HIT = 8 ¥ + 48 bectonres y a Ejgmplo 9. En el cuadrado acjunto, el lado ‘De mide a unidades. Hallar el valor del Angulo @, si P y T son puntos qve trise can los lados del cuadrado. a P Sofucién. Como P y T trisecan a los ladoe % del cuadrado, entonces: 0 a OP=(a,a/3) y OT=(2/3,a) Duego: | |OF|] = []OT|| = Vat+(a/3)? = v70 OP.OT = 8). (8,2) = te2 4 1,2 = 2,2 OP.OT = (2.5). (G3a) ge? + ge 32 OP.OT 2 Si Cosé = —oP.oT > Cosé = 2/3) 2* = 2 }LOP i] | lOT]] (3 70)? -. 8 = arcCos(3/5) Ejemplo 10. Sean & y 6 vectores unitarios en R?, Denostrar que la suma es un vector unitario si y sdélo si el angu lo formado por dichos vectores es de 120°, Demostaacéén. i) Primero denostraremos que | asbp fea En efecto, supongamos que @=120° es el Angulo formado por a y b. Entonces: HleeBl]? = [fap ]2+] [8] p2+28.8 > = Vell?+[ 18] 1242112111181 |cose =1t+i1+ 2(1)(1)(-1/2) = 1 o. [a+b] = 4 ii) Demostraremos que a y 6 forman un Angulo de 120°, En efecto, por hipétesis: Hatl=!]8) [=] [2+6] [= 1 Luego, si |fatb] |? = 1 + [Ja] [2+] /B]]2+22.3 = 4 + 1 44 + 21/811 Bllcose = 1 de donde: Cos® = -1/2 + @ = 129° > Ejemplo- 11. Hallar el valor de r=[[a - 81, si |Ja]]=1. [1b] f=2 y el angulo entre 2 y $ es 60°, Sotucéén, Tenemos: 2.6 = LLELT{]Bf[cosé0° = (1)(2)(1/2) = 14 Vectores 49 Beal lal eae. bel [bE2) = 2. r = 23/3 = Fi 2aeby] +r? ole Ejempio 12. Sean a, 6 y G veclores en R?. Suponer que Héil=1 y [lell=a. si []a-Bee] [=| [a+2bee] 1 y e2 dn- gulo entre a y % mide 7/4; hallar el coseno del d4ngulo entre los + vectores 6 y Ce Sofucién. Tenemos: a.b = Tal EB) lcos(a/4) = (yan 2) = }|a-b+|f2 = | parobedl]? > PLAUPP+ETBEPFet PS] 2+2(-8.6+2.8-8.) = [a] 2ecybl]2+] fel 24 ‘ 2(28. O48. 0420.6) de donde: ] [6] [2422.6+28.% = 0 + 14+ 2(/2/2) + 2]IBL]]{8llcose ++ Cose - - La Ejemplo 13. Por métodos vectoriales, determinar los cosenos de los 4ngulos formados por las aristas y las diagona- les de un paralelepipedo rectangular. So€uczén. Sean a, 6 y @ las aristas y a una de las diagonales del pa- ralelepfpedo rectangular; ademas, sean: a=m°(dya) , B=m°(d,b) , y=m°(d,c) En la figura: ad = vto = arbre + da-+a.a+ a6 + a.c = |fal]? 4.6 = 2.6 4+ 6.64 6.6 = [IB]? G.0 = 2.0 + b.2 + 3.6 = WUll? > +742 > Entorces: Cosa = 2:4 — Hell’ = Hall Hlali idl] lial} [Pop] lid} b.a - __ tll? “Tei an rere ana 50 Vectones Ejemplo 14. En la figura OACB es un para lelogremo. Si 0C=(5,3), BA= (~3,9) y a el angulo determinado por OA y ost, hallar el coseno de a. Solucéén. Si BA=(-3,9) + A-B=(-3.9) (1) OC = OA + AC , pero AC=08 Entonces: OC = 0A + OB + &A+B=(5,3) (2) 7 oS / * De (1) y (2) obtenemos: h=(1,6) y B=(4,-3) B + B+=(3,4) cose < : _ _(1,6).(3.4) | 3424 _ 27 Ejemplo 15. En el paralelogramo ABCD se tiene: [JAB] ]=6, [|AD[]=4 , m(4A)=60°; M es punto medio del lado AB y Nes punto medio del lado BC. Hallar Cos@, sabiendo que: ]/2/|=6 y 13) |=av73. Sotucién. AD = 4(Cos60°,Sen60°) = 2(1,73) -AB = 6(Cos0°,Sen0°) = 6(1,0) Luego: AN = SAB = 3(1,0) y iN = 3AD = (1,¥3) DM = AM-AD = 3(1,0)-2(1,/3) = (1,-/3) Pero: & = rDM > }Jall=rf [DMI] + 6 = r/4¥3 , de donde: r=3 ws a=3(1,-/3) Andélogamente: AN = AB+BN = 6(1,0)+(1,/3) = (7,73) Si b-tan + |[BlJ=t]{AN]| + 4/73 = t7Z943 . de donde: t=2 “ -. 6 = 2(7,73) Por tanto: Cos@ = a = 3(1, -¥3).2(7,¥3) = Ital Het] (37743) (274943) = V73 EJemplo 16. En un AARC se tiene: AC=(-2,4) y AB=(3,-1). Hallar el angulo que forma el vector BC con el vector }?, Solucién. Tenemos: 6-h=(-2,4) y B-R=(3,-1) Restando se tiene: 6-3=(-5,5) + KG=5(-1,1) Cos@ = Bo.t*+_ = 5f-1,1).(0,1) zl > = 6=45° jac] 5/2 42 56 7. Vectorzes 51 EJERCICIOS Hallar la medida del Angulo entre los vectores a y b sia va de A(2,5) a B(4,4) y 6 va de C(3,-2) 0 (2,1). Rp. 6=135° Si ABC es un tridngulo y AC=(4,1), AB=(-4,-3), hallar el co- seno del Angulo que forma el vector BC con el vector unita- rio }=(0,1). Rp. Cos=/5/5 En un tridngulo ABC se tiene: AB=(2/6,272) y AC=(v¥6,-/2). De terminar la medida del Angulo formado por BC y el semieje po sitivo de las abscisas. Rp. 6=120° En un plano cartesiano, los puntos A(r,s), B(natr,nbts) y C(-mbtr,mets) son diferentes del origen y #/0 : n7O. Hallar la medida del dngulo formado por los vectores AB y AC. Rp. 6=90° Hallar el Angulo que forman el vector a que va de A(-1,3) a B(6,4) con ¢l vector b que va de C(5,-1) a D(2,-5). Rp. 6=135° Calcular a.b , donde a y 6 son los b y vectores de la figura adjunta, pa- a ra los cvales: |{af|=8 y [|b] ]=/72 » a Rp. -48 * Calcular |]atb|| sabiendo que a y 6 forman un dngulo de 150° y que: [Jal|=/78 y ||) ]=6 Rp. 273 Sean a, t y @ vectores diferentes de cero, y supuesto que el Angulo entre a y @ es igual al Angulo entre 4 y @ para que valor de t es el vector ¢ perpendicular al vector: a = {|bj]a + tb. Rp. t=-] fall Los vectores a y 6 forman un 4ngulo de 60°, sabiendo que Ilall=5, [6l]=8, determinar: [la+b}l y [la-bIl. Rp. #129 y 7 11. 12. 43. 14 16. 17. 18, > - ~ - o : a Los vectores a y h formen un énguio de 12C , sabiendo gue zy 3 a > - > . Hall=3 y [!Bll=5, detersinar: |}[ar@]} y pfa-Bli. hr. VIG yy 7 : > > . Qué condicién deben setisfacer los vectores a y t para que > . . - + el vector ath bisecte al aéngulo formado por los vectores 2 3 b. , Rp. []atl=118i El vector a=(x,y) se obtiene girando al vector b=({-2,4) 60° en el sentide horaric. Haller el vector é. Rp. a=(2/3-1, 24/3) si [lel l=a y [ibll=b, denostrar que ol vector 2 = atta bise ea el dngulo formade por a y t. Sean a y 6b dos vectores no nulos tales que |fall={[{b]}=m. si el éngulo entre & y 6 es 1/3 redianes, y la norma de su dife rencia es 2-m; hallar nm. Ro. n=i Tres vectores 4, 6 y GeR? satisfacen las Siguientes propieda des: |fall=[[ell=5. PIB] [=1 y {[a-Beep [=] faeBeZ]]. sa et dr- gulo que fornan a y t es u/8, hallar el que forman t ¥ z. Rp. 7/8 Lajos tres vectores no nulos en h?: a, b y ce Supuesto que * > > . > > el angulo que foraan a y ¢ es igual al que forman b y c. De- mostrar que ¢ es ortogonal al vector 1B la- Tal yt. > Los vectores @ y Bb forman entre si un dnguic de 60° y el né- > . . > > dulo de aes 6, Hallar e] méaulo de b pare que a-b forme eon + aun 4ngulo de 30°. kp. 3 En el paralelogramc ABSD se tiene: LD c HABE |=3, [AB] [=6, m(¢a)=40°, P y Q son puntos de triseccién dé los lados AB y Bc respectivaments. Ha- llar Cos@ sabiendo que tfall=27 ¥ I(t l=2v79. Sosg = ZL Rp. Corg A Vectonres 53 1.16 DESCOMPOSICION DE VECTORES Sean los vectores no paralelos a y 6 en R2. Si dese un pun to de vista grafico un vector ¥ del plano pedemos expresarlo co- mo una suma de componentes vectoriales ra y tb, gue son miltiplos escalares de a y $, entonces se dice que se ha efectuado una des composicién del vector ¥ en sus componentes paralelos a los vec- tores a y b (Figura 11). También se dice que ¥ puede expresarse como una combinacéén bi- neaé de los vectores 2 y %, los cuales reciben el nombre de &a- ses del conjunto de vectores veR?. Podemos afirmar entonces que todo vector veR? se puede expresar como una suma de miltiplos escalares de vectores unitarios orto gonales: £=(1,0) y 3=(0,1) En efecto: v= (x,y) = (x,0) + (0,y) x(1,0) + y(0,1) Ge donde: > + +t v=xi + yj Expresién en la cual, los escalares x e y se llaman componentes > + escalares de v paralelas a t y J. Los vectores xi e yi son las componentes vectonriales de v paralelas a i y j (Figura 12). Figura 11 Figura 12 1.17 COMBINACION LINEAL Todo vector a¢R?, puede expresarse mediante una y sélo una combinacién lineal de un par dado de vectores unitarios ortogona > > 2 s : les i y *. Es decir, existe una y sélo una pareja de escalares 5k Vectores s y t tales que: a = su + tut (14) Al multiplicar escalarmente por u se tiene: \° G2 = su.u + tu.y’= s[ful|2+ 0 de donde: dea =s (7) Al multiplicer (14) por i+, se tie ne: i | u > | w.4 = sutb + tut. gt = o +t] atl? su +, + de donde: wea = t (2) Figura 13 Por sustitucién de (1) y (2) en (14) obtenemos: a= G80 + (G4. aGt (15) ~ + También podemos afirmar que el vector a se puede expresar como una suma de nultiplos escalares de vectores crtogonales no nulos que no sean unitarios. > En efecto, si u = 6 4 3 + Het Huth] ofl entonces por (15) se tiene: 2-(_b .2) 8 +( Bit?) b* que equivale a: z= (3:3,)8 +( 2.b* )e* (16) Ejemplo 1. Dados los vectores 4=(-2,2) y 5=(3,1), expreser a co mo una combinacién lineal de 6 y b+. Sclucién. $i B=(3,1) + 6+ = (-1,3) y []BI 1-70 Haciendo uso de la ecuacién (16) se tiene: a = [6227.81 )03,4) 4 [SEP CUD (4,3) (=$42)(3,1) + 248) (1,3) - £(3,1) + 4(-1,3) Verificacién: @ = (- §,-2) + (-4,18) = (-2,2) Vectoazes 55 1.17 PROYECCION ORTOGONAL Sean a y 6 dos vectores y $ no nulo. La proyeccidén ortogo- nal o componente vectorial de 2 sobre b, denotada por Proyza, es el vector: +? . > a Proyza = (==, ye , b#e (7) p HIB) 1? Si aplicamos (17) a (16), obtenemos: 3 = Proypa + Proygia (18) Geométricamente esta definicidn significa que se puede construir . > un tridngulo rectangulo cuya hipotenusa sea el vector 4 y cuyos catetos contienen a los vectores Proyga y Proypee. Figura 14 Figura 15 Propiedades. i) Proys(asb) = Proyza + Proyab li) Proyp(ra) = rProyza Observacién. Los vectores 6 y Proyga son paralelos de tal modo que si el dngvilo 6 entre a y 6 es agudo entonces 6 y Proyga tienen la misma direccidédn y sentido (Fig. 14), en tanto que si © es obtuso entonces % y Proyza tienen la wisma direccidn y sentido opuestos. (Fig. 15) Ejemplo 2. Si a=(12,5) y b=(-3,4), hallar Proyta. Yo€ucién. Segin (17) se tiene: > _ (12,5). (-3,4) 1 Proyta = => s7(-3,4) b (79416)? 5 56 Vectonres Vemos que Proyga y 6 son paralelos y tienen sentidos opuestos en este caso. 119 COMPONENTES ESCALARES > a.b > se denomina componente escafan de a en la Al ninero etl direccién de 6, siendo b no nulo, y se denota por: > > 2.6 Compa = —~> (19) PBT ab \ 6 Toda vez que Proyga = (ene se puede establecer la rela- . - ciédn siguiente entre proyeccidén (un vector) y componente (un nié- mero). ; > > Proyza = (Compya) 6 ee TI Si Compzz>0, entonces la Proyta tiene el mismo sentido de ¢, del mismo modo, si Compga<0 entongces la Proyza tiene sentido opuesto (20) a b. (Fig. 15) Por lo que, podemos afirmar que la componente escalar de un vec- tor es la longitud dirigida u orientada del vector. Esto es, si 6 Heil es un vector unitgrio, la ecuacién (20) se puede escribir: Compza = +| [Proypal | (21) Nota. El signo se debe elegir segtin que b y Proyga tengan o no el mismo sentido. Para los vectores de la Figura 15 se to ma: Comppa = -[]Proygal }. . . > Propiedades, i) Comp? (a+b) = Compa + Comp2b ii) Compz (ra) = rCompga Ejemplo 3. Hallar la proyeccién ortogonal y la componente esca- lar del vector 2=(~3,-4) sobre el vector b=(4,-2) Solucién. Si b=(4,-2) > [1b! [=20, luego segin (17) se tiene: Proy¢a = (Seth 2)) (4,2) => $(4,-2) = 4,3) Vectonges 57 Obtenemos la componente aplicando (19), esto es: > Compga = +2 Set lanl = Sa = - See > > Como la Compza<0, la Proyza y $ tienen sentidos opuestos. Calculando la longitud de la proyeccién: J(475)*4 (2/5)? = 2B | |Proygal | observamos que: Compga -|[Proyzal | Ejemplo 4, Hallar las componentes escalares de a=(-2,2) que son paralelas a los vectores 6=(3,1) yb. Sotueéén, Si b=(3,1) > | [Bb] }=/T0 y b*=(-1,3) zB) by (bY tt erie Granny De la ecuacién (16): a = ( onces: z= { f22+2)-(3,1)]_% ‘(-2,2).(-1,3)] 8+ Entonces: a La ier + eae tii = A + 8) bo ( 70) TTI (Faien e onde s on 22-4 om 2% = De dond c PZ Fs y Cc PF AG Ejemplo 8. Los lados de un tridngulo son los vectores 3, $ y a-b. si |/al|=5, ||6]]=3 y Compga=-5/2, hallar la longitud del ledo a-b. » Soluceén, Si Compza = +5/2 > a.b =-3 > 2.6 = -15/2 Ejemplo .. Los lados de un triaéngulo son los vectores a b y ath. si [lall=s, [[bl]=2”2 y | a+b] |=/53; hallar el valor de 2Coupza-Comp? (at). Sotucién. Si |fatbl[=¥53 + [lal]? + 22.6 + TIBI]? = 53 + (5)?242a.bt(2/Z2)2=53 + &.6=10 58 Vectoses Luego: 2compza = = 2( BB) = 2 = 5/2 Comps(atb) = arb = Lieut + 83 tau. 6.8. ox =7 “. 2ompzé - Compz(atb) = 572-7 Ejemplo 72 Si atbreed=0 , Jlatb]f=e, [ [tl [=b y [Jal] f=c. Haller Compsd . Solucién. Tenemos: arb = -(G4d) > | larbll=[]ted]] + a={[trd]] Elevendo al cuadrado: a = | fe] }7+#2c.d+| a] ]? Entonces: a? = b? + 20.d + b?, de donde; ¢.d = 4(a?-b?-c?) td Hell Luego: Conped = 5ula2-b?~c?) Ejemplo 8. Si el vector % forma un dngulo de 30° con el semieje positive de las X, |[3{{=2, Comppa=-2 y Compp1 &=2/3. Hellar el vector a. Sotucién. % = ||b{|(€0830°,Sen30°) = (¥3,1) Segiin la ecvacién (18): a = Proy¢a + Proypta — t 6+ (¥3,1) (-1,73) +(Compyra) —2— = (-2)'%221? 5 (273) Hep ee 2 2 J. 2 = (-73,-1)#(-/3, 3) = (-2/3,2) Ejemlo 9. Si a=(-2,/12) y b=(-3,73), hallar el fngulo formado por los vectores 2 y Proypia. +3 = (Compga) yok Solucién. Sea: & = Proytia = ( sb Je » Nee Entonces: 2 = [{-2e72). (7, -3) | 75,5) = 475, 3) = 3(4,73) ; (¥3#9) 2 4 ‘ Sean: u]|% y ve > u=(~1,43) y v=(1,79) El éngulo que forman u y ¥ es el mismo que forman a y >> Luego: Cosé = = UV = (1,73). (1,73) = 4 Hull WIVIL | (783) 07F3) 2 “. 8 = 60° Vecionres 5S Ejemplo 10. Si Proyga=(-2,8), Proygsa=(4,1) y $-3+3*, haller la norma de b. . > > > > Solucéén, Si a = Proypa + Proygta * a = (-2,8)4+(4.1) = (2,9) Luego: B = (2,9)+#(-9,2) = (-7,11) “. TBH] = 4770 Ejemplo 1}. Dado el vector a=(-4,2) y Proygia=(-3,3)5 supuesto que Compete es positivo, hallar Compza. Sofucién. Sia = Proyza + Proygsa * (-4,2) = Proyza + (-3,3) de donde: Proyza = (-1,-1) Segin (21): Compza = +||Proyzal | > Compza = + ¥(-1)74(~-1)4% = +72 En la figura se observa que 6 y Proyza tienen sentidos opuestcs, por tanto: Compga = -/2 Ejemplo 12. En la gréfica adjunta, & es un vector unitario tal ques Cotga = 3/3. Si a+¥=8*, hallar Compsé. Sofluceén, Dado Cotga=3”3 y a en el IV cua- drante, entonces: Sena =- a y Cosa = 33 27 27 Luego, si @=(Cosa, Sena) + ts 71(373,-1) Sen75° = Sen(45°+30°) = Sen45°Cos30°#Sen30°Cos45° = B13) Cos75° = Cos(45°+30°) = Cosk5°Cos30°-Sen45°Sen30° = 23-1) + & = [JB] (Co075°,Sen75°) = “3 pS p(73-1,1075) = e(VF-1,75+1) > > luego: v=a wa = r(-¥3-1.73- 1-1 (V9-1.7941) = 2r(-5,-1) $1 (3/3,-1) -2r(-73,-1) 277 2r/3+1 7 > Fer tante: Compec = 60 Vectozes } B c Ejemplo 13. Dado el exdgono regular de lado a, hallar la proyeccién ortogo- nal de FC sobre BE. A D Soduciéa, FC = ||FC]](cos60°,Sené0°) Vv y/ F 4 £E = 2a(1/2,73/2) = a(1,V¥3) b BE = ||BE]|(Cos300°,Sen300°) = za(d ,- Gy FO.BE pp = 2(1.73)-a(1,-Y3).(4, 3) | BE) |? a*(v143)? Ejemplo 14. Un avidn vuela ent sentido del vector a La velocidad del viento es de 50 Kn/m en sentido del vector ¥. Hallar el duplo de la componente de la velocidad del viento en la dirececién del a vion. redertiny = JJ8l] (Cos4s?,sengs°) = [al /Zc1,1) ¥ = 1¥11(cost20°,sen120°) = 50( 1%) = 25(-1,73) YZ, + > 72 25(-1,73). Sllel}] (1,1) Luego: Compzy = “2 = ~ Z = Bare (14/3) Iial| Ila - 2Compr¥ = 25V2(v3-1) Ejemplo 15. Dados les puntos A(-1,3), 5(5,6) y (7,5): si P di- vide al segmento AB on la rasén AP:PB=2, hallar la proyeceién del vector AP sobre el vector BC, Sotuciéna, Sea el punto P(x,y). Si a = 2 > AP = 2PB + (xt1,y-3) = 2(5-x, 6-y) a> | X+1=70-25 + x=3 y-3=12-2y + y=5 Luego: P(3,5) * AP = (3,5)-(-1,3) = (4,2) BC = (7,5)-(5,6) = (2,-1) ! Vecdores 44 Entonces: Proy sp AP = ( AP. BC je = 4,2). (2,-1) {2,-1) 11BC] |? (241)? oo Proyg. AP = §(2,-1) Ejemplo 6. En la figura adjunta se tiene: [lal (=2, 4.3 =/¥2|bI |. Sea u tal que $4t=% y @ el Angulo entre a y %. Hallar Proyza. Sctucién. & = | [8] |(Cos60°,Sen60°) = (1,73) si 2.6 = [[2f/ 1/8} |cose + /2| 1B] | = Hal 1[Bl}cose, de donde: Cosa = Luege: b = [1B] 1 (cos105°, Seni05°) = UB yz ve,vaeve) > t+ = LB ysve, ve-v6) si tieB > a= ttt - Lbs) - 22,78) ( au Je = 28) v2, v6) bray]? r?(/2+6)? = B+ VE 376) = (1,7) Por tanto: Proys = Ejemplo W. En el paralelogramo de la figura se tiene: HE = FC, m({BAD)=60°, La altura relativa a la ba se AD es h. Si M = AB + AB - BD y 3 = Proy api, hellar [|B] ] en funcién de h. ~ . > = Sofucién. Tenemos: M = AB + AE - Pero: A&E = AE + DE, > M = AB + (AD + DE) - (AD - AB) = 253 + DE = 208 4 34 = 3h | > UPL) = WProyggftt| = alo. 3( HBA) - [tal sll eoseo| [|ADI] }}4D] | | ]85]| de donde: |{P]] = FI RBI | En el ADHC: h = ||DC][Sen60° = |J4B]jsenéo° + | {ABI} - 2p , - 52) 2 5 © LBL = 3@Gn) = 2 » 62 Vectonres Ejemplo 18. Sea el cuadrildtero ABCD tal que M(-2,4) y N(4,2) son puntos medios de los lados AB y BC respectiva- mente; DM es paralelo al vector a=(1,4)> (M es paralelo al vec- tor b=(-3,2) y Proy jpDN = 43 (3.2). Hallar ios vértices del cua- drildtero. —= y, ne B! ]Proy—DN : 4 B entonces: AB = r(3,2) DM=t(1,4) + M-B=t(1, 4) B= (-2,4)-t(1,4) (1) AG Dr = N-B = (4,2)-(-2,4)+t(1, 4) wn t gl = De + + = (64+t,-2+4t) ; * . = DN.AB \z Luego, si: Proy jpDN = (Ta) 36 (6+t,-244t) .r (3,2) > $3(3,2) = eee (3,2) 13 r?(¥9+h)? de donde obtenemos: t=2 . Sustituyendo en (1): D=(-4,-4) Como M es punto medio de AB + AB=2MB o sea: r(3,2) = 2(B-) > Be A(3r- 4,248) (2) cM} ]é + H-€ = s(-3,2) + @ = (-2,4)-s(-3,2) = (-2438,4-28) (3) Nes punto medio de BC + W = 3B + 8) Entoneces: 2(4,2) = 5(3r-4,2r48) + (-243s, 4-2s) + (16,8)=(3r+6s-8, 2r-4st16) «» {16 = 3rtés-8 + 3rt6s = 24 8 = 2r-48st16 + r-2s = ~4 Resolviendo el sistema obtenemos: r=2 y s=3 Luego, en (2) y (3). tenemos: B=(1,6) y G=(7,-2) AB=2(3,2) > B-R=(6,4) » K=(1,6)-(6 .4)=(-5,2) fz | | i oy Etemlo 19 L sem wa. OL pecio rectaéngulo en donde: &=(5,12) y &=(-2,3). Hallar su drea. or Solucéén. |fa]| = 452412? = 13 Zz |B] | = comp+sd = Gat _ (-2,3).(-12,5) _ > ° Hall 13 ++ {T4] | = Compsé = 628 _ (-2,3).05,12) _ 5 Be = al) 13 Vectonres 6: Heh) = (lat}-1 lal] = 13-2 = 41 Area del trapecio: S = S(/ lal /+H/BNI) [BI] = 3(13411)3 = 36u? Ejemplo 20. Sean a, beR?-{0)} y r#0. Establecer el valor de ver- dad de las siguientes afirmaciones: > > > a) Proygia = Proy#:b + a=b b) Proy+(Proyga) = Proyg(Proyz6) ~ alld“ 6 [fall=ildtl c) iComps (a*+b) | <{ibll . ree wa d) Si r>0 + Compzta = -Comp za e) Proy,z(ra) = Proyga Sofucién. a) Si Proygia = Proy# B > B* | [Proys. 6 Pero como: Proy2:5| [a> > B|jat + Bla Por tanto, la afirmacién es falsa b) Si Proys (Proyzga) = Proyz(Proyb) , entonces: [ Cee) [er A, ——2--—-|a = | ——-4 Ital}? []o]}!? - > > + \ > + > [ (a8) (8.4) RB = [ (6.4) (8.8) lé (4) St)? } fall? Hall?) i6il? La igualdad (1) se verifica si y sélo si: aeb=0 + a1b6 + aflb™ 4.6 #0, en (1) se tiene: a=b + |[la][=]{B]| Luego, la afirmacién es vendadeac. +> a.(attb ) c) [Compe (2*+3 ){ < J ISI] > sn < [1B1I a aeat + A.B | aa + 2B 6 yey et ‘ [2.8] < RE LSI La afirmacién es veadadene porque se trata de la disigualdad | de Cauchy-Schwertz. +, > a‘.b , > b).a r ad) -Comp.+(at) = - (rb).a" s a rp Hdl] Ir Htl Dedo que: r>0 > {rf=r 7 -Comp,¢(a*) = - 2b 64 Vect ores Pero: at.b = -2.67 y [Jb [=] 16+] | Entonces: -Comp rete") = ae = Compe & Luego, la afirmacién es vendadena.- +) _ [ (ra). (rb) - p{_2sb = : e) Proy pz(ra) = Then rb = air i = rProyga , ¥r#0 La igualdad se cumple solo cuando r=1, por tanto, la afirma- cién es falsa. EjJemplo 21. Sean los vectores &=(k,-2) y B=(2k,k+2), donde keR. Hallar los valores de k de modo que Proyga y & ten- gan sentidos Spuestos . o sea: ut. < 0; pero como ||b[|>0 +» &4.6<0 * * (k,-2), (2k,k+2)<0 2k?-2(k+2)<0 ++ k?2-k-2<0 > (kt1)(k-2)<0 > (k+1<0 nk-2>0) v (k4120 4 k-2<0) > (k<-1A k>2) v (k>-14 k<2) ++ (6 )v (-1 EjJemplo 22. En la figura: TP||Ox, | JOP] |=s8 uf Si Of=n0P+nOP, hallar men Solucién, OP=||OP| | (Cos30°, Sen30°)=( 4/3, 4) Componentes de OT: yx Pero: y = ordenada de OP = 4 + Of=(4,4) Luego: (4,4) = m(4V3,4) + n(-4, 473) > (1,1) = m(V3,1) + n(-1,73) {" = ¥3m -n 1=m + Wn Resolviendo el sistema obtenemos: m = tV3") »n= F/3-1) .. men = 1/8 Ejemplo 23. Se tiene los vectores 2 y $ eon |fa][=2/3. si besarte, calcular el valor de stt. Sotucién. a = | fal 1 (cos60°, Sen60°) Vectonres 65 + 8 = 243(1/2,73/2) = (v3,3) Ordenada de & = Ordenada de a + yr3er-x . b=(-3,3) si B= sa + tat + (-3.3) = s(V3.3) + £63.73) (1) “Usaremos un método més Girecto para calcular s y t. 2 i Multiplicamos escalarnente la ecuacién (1) por (-3,73)": (~3,3).(-¥3,-3) = 9(V3,3).(-¥3,-3) + (0) + 3V3 - 9 = s(-3-9) , de donde: s = f(3-¥3) Multiplicamos escalarmente la ecuacién (1) por (f3,3)*: (-3,3).(-3,73) = s(0) + t(-3,/3). (-3,/73) + 9 - 33 = t(9-3) , de donde: t = 7033) ° = 2 ee stt 5 EJERCICIOS 1. Dados los vectores a y b en R?, demostrar que: Hal]? = (a.8)a + (at. bya 2. Sia y B son dos vectores en R?, demostrar que: Hladi?Bl]? = (a.6)? + (a+.)? 3. Demostrar que: a) Proyz(b-¢) = Proys - Proyzé b) Proy+ (rb) = rProys 4. Sean los vectores a y 6 lados de un paralelograno. Si [jaf] =6 Hall =2|(b]| y Compza=10/3, hallar la longitud de la diago- nal a-b. Rp. 5 5. Dados los vectores a=(¥%,-1) y %=(3,/3), hallar: 2(Proypza + Proyzb) Rp. (3+¥3, 1-73) 6. Sean @ y 6 dos vectores tales que: a=(5,-2), Compzb=- 58 y ~ []B] +29. Hatlar Compga. Rp. -2/29 7. Si Bes un vector del mismo sentido que %=(1,2), tal que: Hlatl=50 y [{b]}=+29. Haller Compza. Rp. -40 66 10. 11. 13. 14. 15 16, 17. 18. 19. 20. Vectones Los lados de un tridngulo son los vectores a. by ba. Si Hall=6, [[bli=2 y [6-2] ]=5; hallar Compga-comsB. Rp. 5/2 Los Tados de un tridnmgulo son los vectores a. 6 y 2-5, [lal |=10, ||}|=6 y Compga=-5. Hallar la longitud de 4-6. Rp. 14 Los lados de un triéngulo son a, by atd, tales que [|a]|=8 ib) |=6 y | [a+b] [=/O8. Hallar: Compe (&+5)-3compz (2-6). Rp. 32 si |fa-d]f=4, [ [bl l=3 y Compe (a-6)=22/3, hallar la norma de a. Rp. V69 si d=atbec, |lall=p, ||Bll=q, |léll=r, a.b=pq, a.tepr y Compze=rs hallar la norma de a. Rp. ptqtr Si atbeé=0, Bee , Jiall=a, |[Bfl=-b, [1S|[=c. Hallar Compza. Rp. ap(e?-22-b?) Si Proyta=(2,-5), Proypta=(-3,2) y b=2ata*. Hallar J1Ol 1. Rp. 57¥2 Sea ||a[|=¥65, |[a+b]|=/76%, comps (a+b) = Te . Hallar > a Compg(a-b). Rp. 12/5 Si a=(5,- 2) y Proygta=(4, 1); hallar Compza sabiendo que Compgia es positivo. Rp. “10 Hallar el dngulo formado por los vectores a y Proypte, si a=(1,2) y B=(1,3). Rp. 45° 3 zy : Los vectores a y de longitudes 2 y 3 respectivamente, for man Angulos de medidas a y & con cl vector e=(4,1). Siendo ° ° 0° St &3(Te TT) (Tre) y Compgs bez, patter 124.1. ap. 10 Hallar el vector } sabiendo que: | lé[|=2¥2, a=(-4,2), Compzé es positivo y Proypia=(-3,3). Rp. (-2,-2) 21. 22. 23 2h. 25. 26. 27. 28. Vectores 67 c D Dado el exdgono regular ABCDEF de la figura, cuyo lado mide 10 unidades y el vector M=BD+FC+BC; hallar: B E [[ProyMl I. Rp. 25 \—y En el paralelogramo ABCD, m({BAD)=60° _—___¢ }{4B]f=a, [ [aD] [=2a, donde aeR-{0}. Si p=] |Proy spac! | y q=||Proy pall, 2 hallar p+q. Rp.e. 58 A BD Sabiendo que: Proya(a, b)=(1,2) y Proy3z(x,y)=(-4.-8), hallar Proy(4a-x, 4b-y). Rp. (8,16) Sea ABCD un recténgulo tal que 2AB=AD 8 E c y [[AB][]=as sean E y F puntos nedios de los lados BC y DC, respectivamente. F Si M = AE+ACHAF, hallar el valor de: ga D Compypit+Com 524 . Rp. (25/2)a A Dado el exdégono regular de lado a, en donde G y H son puntos medios de BC y DE respectivamente; hallar LIxH I, si: B E x = Proy sp ( 5AG)+Proyj,(9AH). Rp. 10a En la figura: a, b y @ son tres vecto res de R? tales que 6 es unitario, 2 z es ortogonal a a y a.b=] fal |(/3/2). Hallar Compa . Rp. 43/2 5 En el reectdngulo de la figura: A F B H, P y Q son puntos medios. AB=/FB, ae OC=4a, OA=a. Si V=HF+AP+QC, hallar: y > + , Compyyv + Compryy. Rp. 56 (26/5+53)a En la figura: [lal|=8, ||6][=6 y > t | (a+6| ]=”68. Haller: Compz(a+b)- 3Compz (a-b). Rp. 32 a Vectores En un trapecio ABCD, los lados paralelos AB y CD miden 9 y 3 unidades respectivamente., Si M es punto medio de AB, N es punto medio de BC y MN=mABtnAD, hallar m-n. Rp. -1/3 30. 31. 32. 33. 3h. 35. 36. En la figura se tiene los vectores y 6, con Hlal|=4. sa $esatta , ha Jar ol valor de stt Rn. 1/2 lar cl valor de stt. Rp / bt py En la figura: a= 30° | OM] [=12. si ON=mOM+nOM*, hallar el valor de mtn. Rp. Z373) Dados los vectores que se muestran en la figura, hallar n+tV3m sabien- +, +a + . + do que: matna*=c , siendo a un vec - > tor unitario y | |e] |=8 Rp. 8/3 En la figura se tiene los vectores > > > a, y ¢, donde |]Ja]|=2/3. si ¢ = ma + nb, hallar mn. Rp. 3/3 En la figura el AABC es equilatero, CH es altura. Si CH=(2,4) y ¥=(V3,1) hallar CompsCA. Rp. 4173/3 En la figura se tiene: a(AOAB)=10u? y | JBl f=. Si Proygia=(x,y), hallar: 4/3xy. Rp. -75 En la’ figura: AB||OY y |JOA][=4. Si OB=n0AtnOA~, hallar el valor de n-n. Rp. (3-3) Vectones 69 1.20 AREA DEL PARALELOGRAMO Y DEL TRIANGULO Haciendo uso Ge la proyeccién ortogonal de un vector sobre otro, estamos en condiciones de hecer otra interpretacién geomé- trica del producto escalar. Para el efecto consideremos el para $+ lelogramo de lados a y ©} (Fig. 16). Llamenos Het] a la altura, que se obtiene mediante la proyeccidén orto gonal de 4 sobre 6 , de modo que: [I |=[ |Proyg+€] |=|Compg.a| = an Recordando que el 4rea del paralelo 6 gramo es igual al producto de su ba Figura 16 se por la altura, se tiene: = HBT = eri | = » pero |/b]|=]167]| + s=]a.b+] en Por lo que, podemos dar la siguienete: DEFINICION 7. El 4rea (S) de un paralelogramo, cuyos lados son los vectores 4 y Bb, es igual al producto escalar de uno de ellos por el ete. del otro. Esto es: = Ja.b+| = |a*.b] (22) En particular. el Area del tridngulo (S81) cuyos lados consecuti- vos son los vectores & y B% esté dado por: > S, = d/a.B*| = 313*.8] (23) Ejemplo 1. Sean A({-3,1), B(7,-1) y C(5,3) tres vértices consecu tivos de un paralelogramo. Hallar su 4rea. Solucién. Tomemos el vértice B como A D punto inicial de los vec- > tores a y %. Entonces: & a= BA = (-3,1)-(7,-1) = (-10,2) Do BR = (25.9) 09..4) = (9 2) S = = o oy \ AImN Gs TG 7624) 5 b L Luego: S = |a.b*| = |({-10,2). ~2)| = 36u? 70 Vectones Ejemplo 2. Hallar el drea del tridngulo de vértices A(-8,-2), B(-4,-6) y C(-1,5). Sofucién. Tomando el vértice A como punto c inicial de los vectores a y t, > _ a se tiene: a = AG = (-1,5)~(-8,-2) = (7,7) & = AB = (-4,-6)-(-8,-2) = (4,-2) Por tanto, segin la ecuacién (23): K B S = 31(7,7)-(4.4)1 = $l28+28] = 280? Ejemplo 3. Hallar el area del paralelogramo sabiendo que sus diagonales estén contenidos en los vectores t= (3, 3) ¥ v=(5,-1). Solucién, En el AABD: ‘A2=6+4+ 74 (1) B C En el AADC: b= a+ 6 (2) 4 a De (1) y (2) obtenemos: ae 4G + ¥) B= 4G - 3) f , D Luego: a=(4,1) y b=(-1,2) + $*=(-2,-1) SiS = [a.B’] + S = |(4,1).(-2,-1)] = ]-8-1] = 9u? Ejemplo 4. Se dan los puntos A(3,-2), B(-3,2) y C(2,7). Si P di vide al segmento BC en la razén BE = 2 ; hallar el a . PC 3’ rea del triangulo APC. Sofucién. Supongamos que P=(x,y) B F c Si 3BP = 2PC Entonces: 3(x+3,y-2) = 2(2-x,7-y) - {8 3y-6 Luego: wor ov a" 4-2x 7+ x=-1 14-2y > y=4 = (-1,4)-(3,-2) = (-4,6) A = (2,7)-(3,-2) = (-1,9) + 6*=(-9,-1) Por tanto: S = 3/a.87] = 4] (-4,6).(-9,-1)| = 15u2 oF Bt al Fl Vectores 71 EJemplo # Sean los puntos A(3,5), B(k.2) y C(5,1). Hallar los valores de k de modo que dichos puntos sean vértices de un triangulo de érea 11u?, Sofuciéa. Tomando A como punte inicial tenemos: B(k,2) & = AB = (k,2)-(3,5) = (k-3,-3) > 6 = AC = (5,1)-(3,5) = (2,-4) S = 318.84] + 49 = 3 (e-3,-3).(4,2)] A 6 de donde: |2k-9[=11 ++ 2k-9=11 6 2k-9=-11 — k=10 6 k=-1 EjJemplo # Los vértices de un tridngulo son A(2,-1), B(4s2) y CeL={ (x,y)/y=x-2}. Si su d4rea es 5u*, hallar la suma de las ordenadas de todos los posibles valores del vértice C. Sofucién. Si C(x,y)eL + C(x,x~-2) Sean: a = AG = €-K = (x-2,x-1) y 6 = &B = BE = (2,3) > S= flab] + 5 = 3) (x-2).(-3,2)] de donde: [4-x]=10 ++ 4-x=10 6 4-x=-10 > x=-6 6 x=14 Luego, hay dos soluciones: C(-6,-8) 6 G(14,12) Por tanto, la suma de las ordenadas es: yity2=4 Ejemplo # En la figura: OACB es un paralelogramo. Si 0C=(5,3) y BA=(-1,5), hallar el drea del tridéngulo OAB. Solucibn. Sean: OA=a y oB=b En el AOBA: a = 6 + BA En el AOAC: OC = 2+ B Del sistema de ecuaciones obtenemos: a =) Luego: a=(2,4) y B=(3,-1) + 6*=(1,3) sis = $fa.b*) + a(aoap) = 31(2,4).(4,20] = 72 72 Vectones Ejemplo #. Hallar el drea del polfgono de vértices en A(-2,3), B(2,7), c(8,2), D(6,-2) y E(2,-5). Sofucién. Dividamos el polfgono en tres tridngulos de dreas §1, So y Ss. Tomando el vértice A como punto inicial de los vectores a 6, t y a, se tienes @ = KB = (2,7)-(-2,3) = (4,4) 6 = AC = (8,2)-(-2,3) = (10,-1) @ = AD = (6,-2)-(-2,3) = (8,- a = AE = (2,-5)-(-2,3) = (4,-8) Si = 1a.b41 = 31(4,4).(1,10)| = 22u? S2 = s1¢.6*] = $1(8,-5).(1,10)| = 21u? Sy = 218.a+] = 3] (8,-5).(8,4)]| = 22u? .. S = Si + Se + Ss =, 65u? Ejemplo 9. En la figura: a(A0AB)=10 , |fal|=5. Si 6=(p,q), hallar el valor de ¥3q+p. | 1&1 | (cos30°, Sen30°) = 3(/3,1) Solucién. a p+ > a(A0AB)=10 + 318*.8] = 10 2(-1,73).(p.q) = — -p + ¥3q = 8 (1) B = [B11 (Cos60°,Sen60°) > (p,q) = “pq? (h,¥8) Por igualdad de las primeras componentes se tiene que: 1 p= 3 Yp*tq? + q = ¥3p (2) Resolviendo (1) y (2) obtenemos: p=4 y qrlv3 -. V3qtp = 16 Ejemplo 10. La figura es un trapecio B Es C . -isésceles, en donder b a=(1,3) y 5=(5,-1). Hallar su érea. F Vectores 73 Solucéén. Sean: ¢ = CE = Proyzsb - Si=a(BCEF) y S2=a(ACED) at. » Bs (Gar [Poa = £(3,-1) Si = [a.¢] = 21(1,3).01,3)1 = 160? glb.et) = yy (s.-1).1,3)] = gua «. S = S,t2S, = 16 + #8 19. 2u? S2 Ejemplo 11. En el tridngule isésceles ABC, hallar: |[|PQ]/+I]PS!]|, si et drea del AABC es 14u? y | [AB] |=] ]BC[{[=2. Sofuceén. Sean: Si=a(AAPB) y S2=a(ABPC) So _ {IBCHIxIIPS|] _ [1PSI} ‘ Si FIABIIx}IPQU] = TYPQH] =A pon G >» Set Si. UPQI+HIPSIT , 44 _ LIPSIT+IIP OHI 1 HIPQq| 1 I1PQi| Fero: Si = S(1{ABIIx/IPQII) = 3(ax! POL!) = 211701 Luego: —4_ . LPSI POUT de donde: ||PS{[+]]PQ]{=7 2] 1PQl] HIPalt Ejemplo 12. En la figura: M=(0,4), N=(5,3), P=(2,-2) y 2(-3,-1) son puntos medios de los la- dos de un trapecio ABCD. Hallar su area sabiendo que | [AB] {=2/5. (8; y Sofuciéa, Por geometria elemental sabe mos que: QN|[AB/|[DC. Dé Luego, siz: QN = N-Q = (5,3)-(-3,-1)=(8,4) Entonces, un vector unitario en la direccién de AM|](2.1) es: a= AM 4 ow = [fama = v3 2D © (2,1) [lanl] YS = + M-A = (2,1) ++ a=(0,4)-(2,1)=(-2, 3) M = A(AtB) + B=2M-A = 2(0,4)-(-2,3) = (2,5) N= 3(BeC) * C=2N-B = 2(5,3)-(2,5) = (8,1) Vectores + D=2P-C = 2(2,-2)-(8,1) = (-4,-5) EB=(2,5)-(-2,3)=(4,2) 3 AC=(8, 1)-(-2,3)=(10,-2) Da=(-2, 3)-(-4,-5)=(2, 8) 2(c#D) Futoncess: . S = a(ADAC)+a(AABC) = 3/DA.AC*| + S]AE.AC*| = Ay(2,8).(2,10)] + $1(2,2).(2.10)) = 56u? Ejemplo 13. Tres vértices consecutivos de un rectaéngulo ABCD Si PeAB, QeCD, los son A=(-8,4), B=(2,-2) y C=(5,3)- ReAD, PQ||a=(7,6) y PQ+PR=(5/3,21/3); hallar el vértice D, puntos P, Qy R, y el 4rea del cuadrildtero PRDQ. Sofuceén. Tenemos: BA=(-8,4)-(2,-2)=2(-§, 3) _ Fero: CD=BA y + D=C+BA=(5,3)42(-5,3) = (-5,9) 7 si Polla + FQ = r(7,6) pF? + Q-P = r(7,6) (1) AP = tBA > P =A + tBA a > P= (-8,4) + t(-5,3) (2) DQ = sCD + Q =D + s(-5,3) A aa% * Q= (-5,9) # 5(-5,3) (3) © Sit) Restando (3)-(Z) obtenemos: x Q-P = (3,5) + (s-t)(-5,3) Luego, en (4): . r(7,6)=(3,5)+(s-t)(-5,3) ++ r(7,6)+(s-t)(5,-3)=(3,5) Multiplicando escalarmente por (5, -3)¢ y luego por (7,6) se tie ne respectivamente: r=2/3 y s-t=-i/3 Si PQ+PR = (3 4 +> FR = 3,4) - (7, 6) = (-3,-4) + R-P = (-3,.-5 (4) 3 Pero: AR=KAD + R = AHKAD = (-8,4)+%(3,5) (5) Restando (5)-(2) se tiene: R-P = k(3,5)-t(-5,3) = (-3,19/3) de donde obtenemos: k=2/3 y t=-1 + s=-1-1/3=-4/3 Por tantc: = (-8,4}-1(-5,3) = (-3,1) = (-5,9) - $(-,3) = (3,5) s R= (-8,4) + 3035) = (-6,44) Area del caadrildtero a (PRDQ) = a(AFRD) + a(APTD) 10. 11. 12. 13. 14. — wn Vectonres a(PRDQ) * Nia Ao |PR.PD*| + 317Q.7D D* 1(-3.42).(-8,-2)1 + $144, 2). (-8,-2)1 = 85/3 u? EJERCICIOS En los ejercicios del 1 al 4, hallar el drea del tridngulo cuyos vértices son los puntos d&dos: A(-5,0) , B(1,3) . C(-3,-2) Rp. S=9u? A(-3,4) , B(6,2) , C(4,-3) Rp. S=24.5u? A(2,-3) , B(4,2) , C(-5,-2) Rp. S=10.5u? A(-1,2) , B(3,5) , €(5,1) Rp. S=1iu? En los ejercicios del 5 al 8 se dan tres vértices consecuti- vos de un paralelogramo, hallar las coordenadas del cuarto vértice y el drea de cada paralelogramo. A(4,-5) , B(-2,3) , C(-3,1) Rp. D(3,-5), S=20u? A(-1,-2) , B(O,1) , C(-3,2) Rp. D(-4,-1}, S=10u? A(-1,-5) , B(2,1) , €(1,5) Rp. D(-2,-1), S=10u2 A(2,4) ,°B(6,2) , C(8,6) Rp. D(4,8}, S=2Cu? En los ejercicios del 9 al 12, hallar el dérea del paralelo- gramo cuyas diagonales son los vectores dados: u = (-2,3)., ¥ = (6,-1) Rp. S=8u2 a= (5,-4) , ¥ = (-1,-8) Rp. seseu B= (11,-1) , ¥ = (=2,4) Rp. S=21x? & = (2,10) , ¥ = (5,-2) Rp. S=27u? En los ejercicios del 13 al 15, hallar el Area de los polige nos cuyas coordenadas de sus vértices son: A(2,5) , B(7,1) , C(3,-4) y D(-2, 3) Rp. S=39.5u2 A(1,5), B(-2,4), c(-3,-1), D(2,-3) y E(5,1) Rp. S=40u? 76 15. 16. 17. 18, 19. 20. 21. 22. 23. Vectores A(-5,-2), B(-2,5), C(2,7), D(5,1) y E(2,-4) Rp. S=66u? Dados los puntos A(2,-1), B(-2,3) y C(4,3). Si P(x,y) divide al segmento BC en la razén BP:PC=-2:5, hallar el drea del triaéngulo PCB. Rp. S=10u? Dados los puntos A(-3,~-5), B(3,1) y C(2,5). Si P(x,y) es el punto de triseccién, m&s cercano de A, del segnento AB, cal- cular el drea del tridaéngulo PCE. Rp. S=10u? Los vértices de un tridngulo son A(3,-1), B(1,k) y C(5,2). Hallar la ordenada del vértice B sabiendo que el drea del tridngulo es de 6u?, Rp. k=2 6 k=-10 En la figura: yUs OABC es un paralelogramo. Si OB=(1,6) A y AC=(9,-2), hallar el drea del tridén gulo ABC. Rp. S=14u? 0 x Los vértices de un triangulo son A(3,-5), B(2,5) y C pertene ce a L={(x,y)/y=-2x}. Si su 4rea es de 3.5u”, hallar las co- ordenadas del vértice C. Rp. C(4,-8) 6 6(9/4,-9/2) En la figura: a(AOAB)=15u2 y |[a]|=10. si b=(m,n) hallar el valor de: 3mtn. Rp. 0 Los vértices de un triangulo son A(x,y), B(4,3) y C(-2,6). Si el 4rea del tridngulo es de 9u? y AcL={(x,y)/x-2y=4}, ha- llar las coordenadas del vértice A. Rp. A(10,3) 6 a(4,0) En la figura: a(AOAB)=12u2, [|] {=2/Z. si Proyg+a=(x,y),hallar el valor de: A y Vio a t Xey. - y Rp. 36 \ (15° 0 x Vectionres 77 1.21 DEPENDENCIA LINEAL Se dice que des vectores a y beR*, son €ineatmente depen- dientes si_uno de ellos es miltiplo escalar del otro; es decir, . > t 6 tera si a=rbt 6 bsra para un escalar r. - > = : = En consecuencia, a y 6 son linealmente dependientes precisamente cuando 2 y 6 son colineales. (Fig. 17) Figura 17 (Vectores linealmente dependientes) 1.22 INDEPENDENCIA LINEAL > . Se dice oue dos vectores ay ber?, son €ineatmente indepen . es : > a = dientes si y sédlo si ay 6 no son linealmente dependientes . es > = = to es, cuando los vectores a y 6 no son colineales. (Fig. 18). ~~ + « a . > —~ b A Figure 18 (Vectores linealmente indeperndiertes) 1.23 CRITERIO DE INDEPENDENCIA LINEAL > . . . . Dos vectores a: F *?, son linealmente independientes si se verifican las condicione siguientes: Si: sa +tB=6@ + s=0 y t=0 (24} PROPOSICION 1.4 Dos vectores & y 6 son Linealmente independien- tes si y sédlo si a/(b. > > Demostauceén. (+) Demostrarenos primerc que si ab entcnces @ y % son lineelmente independientes. > > En efecto, supongarmos que alA% y que satth=6. Al dividir ambos mienbros de esta igualdad entre s 6 t, se tiene 78 Vectones é 6b =rb 6 §= ka Por lo que: allt (& y B son linealmente dependientes) lo que contradice ia hi pétesis. . - (Pa Esto es: > . . En consecnencia, A ¥ % son linealmente independientes. (+) Dewostraremos que si 2 y % son linealmente independientes en tonces: aV\b. . En efecto, supongamos que allt, ae ¥y t#e + aréC/e = rb lo que significa ques a+ (-r)b = @- Se ha logrado una combinacién lineal de a y % igusl a @ econ coeficientes 1 y -r que son diferentes de cero, lo cual con- tradice la condicién (24). Esto significa que a y b son li- nealmente dependientes, lo que contradice nuevamente la hipé tesis. En consecuencia: alt. 1.23 REGLA DE COMPARACION DE COEFICIENTES Sean a y 6 vectores linealmente independientes para los cuales se cumple: sa + tb = na + nb y que se puede expresar como: (s-m)a + (t-n)é = 0 Segin la ecuacién (24) ocurre que: s-m=0 y t-n=0, esto es: s=m y t=n, por lo que podemoe afirmar que: Si a y b son linealmente independientes, y si: sa + tb = ma + nb + { t (25) rn Ejemplo 1. Hallar los valores de k para que los vectores:: a=(-7,k+2) y b=(1-2k.1) sean linealmente independien tes. Solucién. Sabemos que dos vectores a y % son linealmente depen- dientes + al|b , o bien: a.b6*=0 Entonces: (-7,k+2).(-1,1-2k)=0 > 74#(k+2)(1-2k)=0 Vectores 79 de donde: 2k743k-9=0 > k=-3 6& k=3/2 Luego, a y $ son linealmente independientes si y sélo siz k#-3 y k#3/2, esto es:- keR-{-3, 3/2} Ejemplo 2. Sean a y % vectores linealmente independientes. Para qué valores de k tendremos que ¢=3a-26 y d=kat4b son linealrente independientes. Sofuceén. Debemos hallar nimeros s y t, que no sean simultdnea- mente cero, de modo que: s(3a-26)+t(kat4b)=8 «+ (38+t)a + (4t-2s)b = 6 Por la ecuacién (24), 1a independencia lineal de a y % implica que: 3stkt=0 y 4t-2s=0 De la segunda ecuacién se tiene: s=2t, y en la primera ecuacién implica que: 6ktkt=0 + t(6+k)=0 ++ t=0 6 k=-6 Pero como t y s no son ambos cero, entonces los vectores e y a son linealmente independientes si k=-6. PROPOSICION 1.5 (Teorema de las Bases). Si a y 6 son veetores linealmente independientes del plano, entonces a y % forman una base de los vectores del plano. Denostaccién, Sean a=0Q, b=OR y c=0P Por hipétesis a y 6 son linealmente independientes, entonces OQ y OR no son colineales. Por P trace mos paralelas a 0Q y OR de modo que i @ ii tercepten a sus prolongaciones en M N respectivamente (Figura 19). Luego se tiene: ON=sa y onN=+b Pero como OP=ON+NP=ONtOM, entonces: Figura 19 @ = sa + tb _lo que nos permite afirmar que @ se representa como una fnica combinacién lineal de a yb y genera el espacio vectorial R?. En sintesis podemos decir que, dado dos vectores a y 6 en R?, en tonces: at ++ {2,56} es una base del espacio R?, La demostracién anterior nos sugiere Ja siguiente definicién. 80 Vectones > . DEFINICION 7. Dos vectores a y $ constituyen una fase de los vectores del plano -si, todo vector c del plano se de manera inica como una combinacién lineal de a puede expresar y_b. Es decir: > + 3 y B generan aR? ++ VeeR?, As,teR/ C=sattb . > En efecto, al multiplicar 1a éltima igualdad por a* y 6+ ccurre +, Z, > t- Be 6) be be >), > que: at.e $.% = s(b+.4) > ss 6.8 a. + 1, > b’.e\> av .c\> : = s+} (26) Por tanto: c= (a3) + (==) Observaciones. (1) Un vector no nulo se puede expresar no sola- , mente como una combinacién lineal de dos vec tores ortogonales a y at, sino que 2+ se puede reemplazar porsuaiquier otro veetor que cumpla la condicién de no ser paralelo a Be (2) Los némeros s y t de la ecuacién (26) se denominan coondena- das del vector @ en la base a=(4, bd}. La notacién (26), se denomina, ademas, descomposicién del vector ¢ segin la base B={a, bd}. (3) A manera de una generalizacién podemos decir que: - > > > > . Un conjunto de vectores {81,a2,a3, cceee an} de un espacio n . . : vectorial R” es una base para este espacio vectorial si se cumplen las condi ciones siguientes: a) 81,82,43, weeny son linealmente independientes. b) 21,82,855 ck generan el espacio vectorial R™, Si el vector A es una combinacién lineal de los vectores ai, Bos ween dys con coeficientes AasAay soeendys es decir: n a= > Ay by k=1 entonces-cada coordenada X. ,(@) del vector & es igual a la su ma de los productos de los” coeficientes Aaphereses dns por las coordenadas homéninas ‘de los vectores B1sGarceesdys Vectores 87 s n Eetc es: x, (a) - > AX, (8) » TANT, 2, Be veee K=1 DEFINICON 8. Se denomina proyeccién del vector @ sobre el vec- > = = tor @ segin la direccién 6 al vector: > B18 \> P -({P+e 7 Foy (3,8)° (a )a (27) Anlicando esta definicién a la ecuacién (26), ocurre que: > > | > ~ c= Proy (3 8) * Proy (g 3)¢ (28) Ejemplo 3. Expresar el vector c=(4,-5) como combinacién linesi de los vectores a=(-2,3) y b=(3,-1). Sofucién. Hallemos las coordenadas (s,t) de ¢ segin la base {a,b}. Aplicando (26) se tiene: >, > B.S _ (4,3).(4.-5) 2.11, g 2 Bee _ (-3,-2). (4-5) 2 2 +3 . (-3,-2).(3,-7) wl 28 (1,3). (-2, 3) e » 11 2 w & 2-23) + 2G3,-1) Ejemplo 4. Si G=(4,-5), a=(-2,3) y b=(3,-1), hallar Proy 2 tye y Proy¢ ze y verificar la ecuacién (28) Sofluceén. Utilizando los resultados del ejemplo anterior se tie ne? Proy* t)é = - H(-2, 3) y Proy(g xe = £(3,-1) En consecuencia: @ =- Ui-2,3) + $(3,-1) = (4,-5) Ejemplo 5. Sean & y Bb vectores linealmente independientes y co- mo tal, susceptibles de formar una base. Demostrar > + > 2 que c=3at2b y d=2a-56 también forman una base. Demostaacéén., En efecto, verificaremos que e y a son linealmen- te independientes, aplicando (24). Si so + td = @ + s(3a-26) + t(2a-5b) = 0 > > > (3s-2t)a + (2s-5t)b = @ Pero por hipdétesis, a y 6 son linealmente independientes; luego, 82 Vectoses aplicando de nuevo (24) se tienes 3st2t=0 y 2s-5t=0. La resolucién del sistema nos da: s=t=0 Por tanto, ¢ ¥ d son linealmente independtentes. Ejemplo 6 Fijado el vector @ en g?, entonces tes expresable en forma Unica, como la combinacién lineal de los siguientes pares de vectores: (1) a=(-3,2) y t(-2, 3) (3) 8=(3/5,1) y B=(-1,5/3) (2) a=(2/3,1/5) y B=(-1,-3/10) (4) &=(¥2/3,4/3) y b=(3/2, 372) Establecer el valor de verdad de cada afirmacién,. Sotucién. Sabemos que: ¥ceR?, 3s,teR/ c=satth allt Veremos entonces si cada par de vectores dados son paralelos. (1) B= xB > (-3,2) = v(-2,3) $F -2F + rH3/2 2=3r + r=2/3 Luego, #!reR tal qus a=rb > ait -. Es verdadera (2) f=rb + (2,4) = r(-1,-<3) 2/as-r + rv=-2/3 esr 5° 76 1/5 = (-3/10)r + r=-2/3 Luego, 3!reR tal que a=rb + af|[% .. Es falea (3) Z- 2b > 2.1) . r(-1,4) - {7 Ser + pen3/5 1 = (5/3)r + r=3/5 Luego, JIreR tal que a=rb + ayit -. Es verdadera (4) a.b* = (72/3, 4/3). (-3/2, 3/2) = -242 = 0 sia.b’=0 + 21/8 .. Es falsa. Ejemplo 7. Sean {41,22}, ({by,b 2} bases de R? y a=26;-3b2. Si @i=bi-2b2, 2 ag= =3b1+(1/2)b, y a= =maj+nae, hallar el va Lor de a-n. Solucién. Si a1=62-2b, > $1=41126, (1) az = 3(a142b62) + $b, de donde: 8, = - 7S + gas Sustituyendo en (1) obtenemos: %, = TE + 1 + 74ae - Fe. i> > Entonces: a = 2(qq81 + qfia)-36 7 + qa.) = eh + Te Vectones 83 “4 co ._ > > +> Luego, si a = m@; + nao * o-n = 1 Wl Ejemplo 8. Halle las férmulas del cambio de base, siendo Uy=vievas Ga=3vi-35V2. y determine las coordenadas del vector U respecto de la bese B'=(¥1,¥2), si respecto de la base B=(U;,U2) son (2,-1). : : : : + + SofucéSén. Resolviendo el sistema de ecuaciones para v: y v2 ob- > > 1+ + 3> > 2 = = - = = = ~~ = tenemos: vi Sui puz os V2 guy pu2 Si (2,-1) son las coordenadas de & respecto de la base B=(U1, U2) entonces: U = 2ui-u2. Sean (s,t) las coordenadas de U respecto ds BY=(¥i,v2) + u = svi + t¥2 = s(3t, - #2) + (Zu, - Fiz) + 2 - U2 = 3(5st3t) x - F(s+t)ie Segén (24): 2 = 3(5st3t) + 5et3t=4 t= Z(stt) + stt=2 De acnde obtenemos: s=~-1 y t=3. Luego, (-1,3}) son las coordene- das de u respecto de la base B'. Ejemplo 9. El vector p=(-5,2) se descompone en pil/x y pally. El vector q=(2, 1/2) se descompone en ailix y aal ly. > > > + Si x=(2,1) e y=(-2,-5): hallar el valor de (Bitq1). (D2ta2). Scéucién. Sea: P = ox + ny § = 2m-2n » (-5.2) = n(2,1)4n(-2,-3) f- | 25 n- 3n de donde: m=-19/4 y n=-9/4 + pr =- Bl2,1) y pe =- 4(-2,-3) Si ¢ = rk+t} + (2,1/2) = r(2,1)+t(-2,-3) «+ J 2 = 2Pr2t 1/2 = r-=3t de donde: r=5/4 y t=1/4 + qa = 3(2,1) y q2= F(-2,-3) Por tentor (pitqi).(p2td2) = (--4)(-2) (2.1). (-2,-3) = 2-4-3) = -49 84 Vectones B Ejemplo 10. En el tridngulo ABC se tiene: AM:NG=3:4. Si BM=rBA+tBC, ha- ilar el valor de rtt. Sofluceéda. En cl AABM: A 3M 4 c Si BM = rBA + tBG + r=4/7 y t=3/7 “. rtt=4 Ejemplo 11. En la figura se tiene el para lelogramo ABCD. Si P es punto medio de GB, QD=7QB y si PQ se escribe co- mo una combinacién lineal de DC y AD, cal- cular la suma de los escalares. Soéucién. Sea PQ = sDC + tAD (1) En el AQBP: PQ = PB-QB , pero: PB = 308 » QB = 30 : pO- 2m - ig -i¢m 4(355) - i 4;_ip Entonces: PQ = 5CB - 4QD = o(-AD) - 5(gBD) = AD - 9(-DB) = 1np + A¢iR-m) = Jao AD = gAD + g(AB-AD) = gDC - 24D Segin (1): sDC + tAD - gC - ZAD ++ 5-1/8 y t=-5/8 F c Ejemplo 12. En el paralelogramo de la fi y gura: AE = tag, DR = 455. / s / Si EF=mAB+nAD, hallar el valor de n-n. L E as / Solucéén. En el cuadrildtero ADFE: A B EF = EA + AD + DF = -Ke + AD + 00 =-4aG + a+ lin -1G8 4 5) + + Is 7 (AB + BC) + AD + BA . - ew - 1m: 3m - pee Como BC = AD + EF = JAB + 4 = mAB + nAD 4+ m=1/4 y n=3/4 “. n-m = 1/2 Vecfores 85 Ejempio !3. Er e? triangulo ABC, las lon gituces de los segnentos BD yx DS son 3 y © respectivamente. Si AD=mAB+nAC, hallar el valor de mtn. — — A Sofuceén. En el AABD: AD = AB + BD - = AB + 2Bc = AB + 2(AG-AB) = 2AB + OAC D AB + GBC = AB + (AC AB) = gAB + GAC Luego, si: nAB + nAC = 245 + ZAC “ff = 5/8 on = 3/8 .. mtn = 7 Ejemplo 14 Se tiene el cuadrilatero ABCD. B Sabiendo que AE = 5B yFyG son puntos de triseccién de CD y M es pun- F to medio de EF. Al expresar AM como una con CG binaciém lineal de AB, BC y CD, hallar la suma de todos los escalares. A Soflucién., Seas AM = wAB + nBC + rCD En el AAEM: AM = AE + BM 35 + 3aF - 4a le Ht = GAB + 5(EB + BC + CF) = 155 + 12m + Be CL = GAB + 3 (GAB + BC + 3D) Luego, Si: wAB + nBC + rCD = 218 + aC + ze + (m-2/3)AB + (n-1/2)BC + (r-1/6)CD = 6 Como AB, BC y CD son linealmente independientes, entoncee: n-2/3=0 , n-1/2=0 , r-i/6=0 «+ m=2/3, n=1/2 , r=i/6 . Bintr = 4/3 Ejempio 15. En el paralelograno adjunto, B P G P y Q son puntos medios de BC y AE respectivamente RD=3AR. Si RC se ex- Q presa como una combinacién lineal de PQ y PA, hallar el productc de los escalares. Setueién. Sea RC = mPQ + oFA 86 Vectonres ma a i En el ARDC: RC Bee os ee " hy ? oO — = Pa) = -Po . Iba + 3(PQ -~ PA) = -PQ SPA Luego, siz mPQ + nPA = -PQ - SPA —- m=-T y n=-1/2 . mn = 1/2 Ejemplo 16. Sea ABCD un paralelogramo, M un punto sobre el lado BC. Si el drea del AABM es igual a la mitad del 4- rea del cuadrildtero AMCD y AM=sDC+tAD, hallar el valor de st+3t. Sofucién. Si a(AMCD) = 2a(AABM) ye Ooo > a(ABCD) = 3a(AABM) | Entonces: (BOjh = 2(BM)h ++ BM = $5 h ‘Luego: AM = AB + BM = DC + ay oi sDC + taD = DC + $id > s=1 y t=2/3 -. st3t = 3 Ejemplo 17. En el paralelogramo ABCD se B G » AE. 1 AP cumple: ED = n-1 Y ‘ac Si M=mAD-nAk, probar que: M = AB, -1 a A E Demosinacién. En efecto: AB = AC-BC = AC-AD = AC-(AE+ED) = AG-AE- Pero: AC = mAD y ED = (n-1)AE ED Entonces: AB = mAP-AE-(n-1)AE = mAP-AF-nAF+AE “. AB = mAP - nAE = 6 Ejemplo 18. En la figura: ABC es un tridn- gulo equiléterc. Si AB=nAC-nHB donde H es el ortocentro, hallar el valor 1 de: 4 t=, n 2D

S-ar putea să vă placă și